Multiple Choice Questions PROB. 1-1 (AICPA) : Chapter 1 - Partnership

You might also like

Download as pdf or txt
Download as pdf or txt
You are on page 1of 72

Chapter 1 - Partnership

MULTIPLE CHOICE QUESTIONS

PROB. 1-1 (AICPA)

Which of the following is not a characteristic of most partnership?

a. Limited liability
b. Limited life
c. Mutual agency
d. Ease of formation

PROB. 1-2 (AICPA)

Which of the following is not a characteristic of the proprietary theory that influences
accounting for partnerships?

a. Partners' salaries are viewed as a distribution of income rather than a


component of net income.
b. A partnership is not viewed as separate entity, distinct, taxable entity.
c. A partnership is characterized by limited liability.
d. Changes in the ownership structure of a partnership result in the dissolution of
the partnership.

PROB. 1-3 (AICPA)

Which of the following statements is correct with respect to a limited partnership?

a. A limited partner may not be an unsecured creditor of the limited partnership.


b. A general partner may not also be limited partner at the same time.
c. A general partner may be a secured creditor of the limited partnership.
d. A limited partnership can be formed with limited liability for all partners.

PROB. 1-4 (AICPA)

An advantage of the partnership as a form of business organization would be

a. Partners do not pay income taxes on their share in partnership income.


b. A partnership is bound by the act of the partners.
c. A partnership is created by mere agreements of the partners.
d. A partnership may be terminated by the death or withdrawal of a partner.

PROB. 1-5 (AICPA)

When property other than cash is invested in a partnership, at what amount should
the noncash property be credited to the contributing partner's capital account?

a. Fair value at the date of contribution.


b. Contributing partner's original cost.
c. Assessed valuation for property tax purposes.
d. Contributing partner's tax basis.

PROB. 1-6 (Adapted)

A and B formed a partnership, each contributing non-cash assets into the partnership.
Partner A contributed inventory with a current market value in excess of its carrying
amount. Partner B contributed fixed asset with a carrying amount in excess of its
current market value. At what amount should the partnership record each of the
assets contributed?

Inventory Fixed Asset


a. Carrying amount Market value
b. Market value Carrying amount
c. Carrying amount Carrying amount
d. Market value Market value

PROB. 1-7 (Adapted)

Partnership capital and drawings accounts are similar to the corporate

a. Paid in capital, retained earnings, and dividends accounts.


b. Retained earnings account
c. Paid in capital and retained earnings accounts.
d. Preferred and common stock accounts

PROB. 1-8 (Adapted)

On April 30, 2016, Al, Ben, and Ces formed a partnership by combining their separate
business proprietorships. Al contributed cash of P50,000. Ben contributed property
with a P36,000 carrying amount, a P40,000 original cost, and P80,000 fair value. The
partnership accepted responsibility for the P35,000 mortgage attached to the
property. Ces contributed equipment with a P30,000 carrying amount, a P75,000
original cost, and P55,000 fair value. The partnership agreement specifies that
profits and losses are to be shared equally but is silent regarding capital contributions.
Which partner has the largest capital account balance at April 30, 2016?

a. Al
b. Ben
c. Ces
d. All capital balances are equal

PROB. 1-9 (Adapted)

Al, Sharif, and Booba formed a partnership. Al will contribute cash of P50,000 and his
store equipment that originally cost P60,000 with a second-hand value of P25,000.
Sharif will contribute P80,000 in cash. Booba, whose family sells computers, will
contribute P25,000 cash and a brand new computer that cost his family's computer
dealership P50,000 but with a regular selling price of P60,000. They agreed to share
profits and losses equally. Upon formation, what are the capital balances of the
partners?

Al Sharif Booba
a. 75,000 80,000 85,000
b. 80,000 80,000 80,000
c. 88,333 88,333 88,334
d. 110,000 80,000 75,000

PROB. 1-10 (Adapted)

On January l, 2016, Atta and Boy agreed to form a partnership contributing their
respective assets and equities subject to adjustments. On that date, the following

Atta Boy
Cash 28,000 62,000
Accounts receivable 200,000 600,000
Inventories 120,000 200,000
Land 600,000
Building 500,000
Furniture & fixtures 50,000 35,000
Intangible assets 2,000 3,000
Accounts payable 180,000 250,000
Other liabilities 200,000 350,000
Capital 620,000 800,000

The following adjustments were agreed upon:


Accounts receivable of P20,000 and P40,000 are uncollectible in A's and B's
respective books.
Inventories of P6,000 and P7,000 are worthless in A’s and B’s respective books.
Intangible assets are to be written off in both books.
What will be the capital balances of the partners after adjustments?

Atta Boy
a. 592,000 750,000
b. 600,000 700,000
c. 592,000 756,300
d. 600,000 750,000

PROB. 1-11 (Adapted)

Mary admits Jane as a partner in the business. Balance sheet accounts of Mary just
before the admission of Jane show: Cash, P26,000, Accounts receivable, P 120,000,
Merchandise inventory, P180,000, and Accounts payable, P62,000. It was agreed
that for purposes of establishing Mary's interest, the following adjustments be made:
l.) an allowance for doubtful accounts of 3% of accounts receivable is to be
established; 2.) merchandise inventory is to be adjusted upward by P25,000; and 3.)
prepaid expenses of P3,600 and accrued liabilities of P4,000 are to be recognized.

If Jane is to invest sufficient cash to obtain 2/5 interest in the partnership, how much
would Jane contribute to the new partnership?

a. 176,000
b. 190,000
c. 95,000
d. 113,980

PROB 1-12 (AICPA)

Roberts and Smith drafted a partnership agreement that lists the following assets
contributed at the partnership's formation:

Contributed by
Roberts Smith
Cash P 20,000 P 30,000
Inventory 15,000
Building 40,000
Furniture & equipment 15,000

The building is subject to a mortgage of P 10,000, which the partnership has


assumed. The partnership agreement also specifies that profits and losses are to be
distributed evenly. What amounts should be recorded as capital for Roberts and
Smith at the formation of the partnership?

Roberts Smith
a. 35,000 85,000
b. 35,000 75,000
c. 55,000 55,000
d. 60,000 60,000

PROB. 1-13 (AICPA)

May 1, 2016, Cobb and Mott formed a partnership and agreed to share profits and
losses in the ratio of 3:7, respectively. Cobb contributed a parcel of land that cost him
P 10,000. Mott contributed P40,000 cash. The land was sold for P18,000 on May 1,
2016, immediately after formation of the partnership. What amount should be
recorded in Cobb's capital account on formation of the partnership?

a. 18,000
b. 17,400
c. 15,000
d. 10,000

PROB. 1-14 (AICPA)

The Grey and Redd Partnership was formed on January 2, 2016, Under the
partnership agreement, each partner has an equal initial capital balance Partnership
net income or loss is allocated 60% to Grey and 40% to Redd. To form the
partnership, Grey originally contributed assets costing P30,000 with fair value of
P60,000 on January 2, 2016, and Redd contributed P20,000 cash. Drawings by the
partners during 2016 totaled P3,000 by Grey and P9,000 by Redd. The partnership
net income in 2016 was P25,000.

a. Under the goodwill method, what is Redd's initial capital balance in the
partnership?
a. 20,000
b. 25,000
c. 40,000
d. 60,000

b.Under the bonus method, what is the amount of bonus?


a. 20,000 bonus to Grey
b. 20,000 bongs to Redd
c. 40,000 bonus to Grey
d. 40,000 bonus to Redd

PROB. 1-15 (AICPA)

Abel and Carr formed a partnership and agreed to divide initial capital equally, even
though Abel contributed P 100,000 and Carr contributed P84,000 in identifiable
assets. Under the bonus approach to adjust the capital accounts, Carr's unidentifiable
asset should be debited for

a. 46,000
b. 16,000
c. 8,000
d. 0

PROB . 1- 16 (Adapted)

On April 30, 20165 Alex, Benjie, and Cesar formed a partnership by combining their
separate business proprietorships. Alex contributed cash of P500,000. Benjie
contributed property with a P360,000 carrying amount, a P400,000 original cost, and
P800,000 fair market value. The partnership accepted responsibility for the P350,000
mortgage attached to the property. Cesar contributed equipment with a P300,000
carrying amount, a P750,000 original cost, and P550,000 fair value. The partnership
agreement specifies that profits and losses are to be shared equally but is silent
regarding capital contributions.

What are the capital balances of the partners at April 30, 2016?
Alex Benjie Cesar
a.500,000 800,000 550,000

b.500,000 450,000 550,000

c.500,000 360,000 300,000


d.500,000 400,000 750,000

PROB. 1 - 17 (Adapted)

On January 2, 2016, Abel, Cain, and Josuah formed a partnership. Abel contributed
cash of P100,000 and a delivery equipment that originally costs him P 120,000, but
with a second hand value of P50,000. Cain contributed P 160,000 in cash. Josuah,
whose family sells office equipment, contributed P50,000 in cash and office
equipment that cost his family's dealership P 100,000 but with a regular selling price
of P120,000. In 2016, the partnership reported net income of P120,000.
On December 31, 2016, what would be the capital balance of the partners?

Abel Cain Josuah


a. 257,500 200,000 192,500
b. 190,000 200,000 210,000
c. 260,000 200,000 190,000
d. 187,500 200,000 212,500

PROB. 1-18 (AICPA)

The Flat and Iron partnership agreement provides for Flat to receive a 20% bonus on
profits before bonus. Remaining profits and losses are divided between Flat and Iron
in the ratio of 2:3, respectively. Which partner has a greater advantage when the
partnership has a profit or when it has a loss?

Profit Loss
a. Flat Iron
b. Flat Flat
c. Iron Flat
d. Iron Iron

PROB. 1-19 (Adapted)

Partners A and B share profits and losses equally after each has been credited in all
circumstances with annual salary allowances of P30,000 and P24,000, respectively.
Based on this agreement, in which of the following circumstances will Partner A
benefit by P6,000 more than PartnerB?

a. Only if the partnership has net income ofP54,000 or more for the year.
b. Only if the partnership does not incur a loss for the year.
c. In all earnings or loss situation.
d. Only if the partnership has earnings of at least P6,000 for the year

PROB. 1-20 (AICPA)

Downs, Frey, and Vick formed the DFV general partnership to act as manufacturer's
representatives. The partners agreed Downs would receive 40% of any partnership
profits and Frey and Vick would each receive 30% of such profits. It was also agreed
that the partnership would not terminate for 5 years, After the fourth year, the partners
agreed to terminate the partnership. At that time, the partners capital accounts were
as follows: Downs, P20,000; Frey, P 15,000; and Vick P10,000. There also were
undistributed losses of P30,000. Vick's share of the undistributed losses will be

a. 0
b. 1,000
c. 9,000
d. 10,000

PROB. 1-21 (AICPA)


The partnership agreement of Reid and Simm provides that interest at 10% per year
is to be credited to each partner on the basis of weighted-average capital balances. A
summary of Simm's capital account for the year-ended December 31, 2016, is as
follows:

Balance, January 1 P140,000


Additional Investment, July 1 40,000
Withdrawal, August 1 15,000
Balance, December 31 165,000
What amount of interest should be credited to Simm's capital account for 2016?

a. 15,250
b. 15,375
c. 16,500
d. 17,250

PROB. 1-22 (AICPA)

Partner Ae first contributed P50,000 of capital into existing partnership on March 1,


2016. On June l, 2016, said partner contributed another P20,000. On September l,
2016, he withdrew P 15,000 from the partnership. Withdrawal in excess of P 10,000
are charged to the partner's capital accounts. What is the annual weighted average
capital balance of Partner Ae?

a. 32,500
b. 51,667
c. 60,000
d. 48,333

PROB. 1-23 (Adapted)

If the partnership agreement does not specify how income is to be allocated, profit
and loss should be allocated

a. Equally.
b. In proportion to the weighted average of capital invested during the period.
c. Equitably so that partners are compensated for the time and effort expended
on behalf of the partnership.
d. In accordance with their capital contribution.

PROB. 1-24 (Adapted)

Which of the following is not a component of the formula used to distribute income?

a. Salary allocation to those partners working.


b. After all other allocation, the remainder divided according to the profit and loss
sharing ratio.
c. Interest on the average capital investments
d. Interest on notes to partners.

PROB. 1-25 (Adapted)

Which of the following is not considered a legitimate expense of a partnership?


a. Interest paid to partners based on the amount of invested capital.
b. Depreciation on assets contributed to the partnership by partners.
c. Salaries for management hired to run the business.
d. Supplies used in the partners' offices.

PROB. 1-26 (AICPA)

The fact that salaries paid to partners are not a component of partnership income is
indicative of
a. 30 12 18 5 A departure from generally accepted accounting principles.
b. Being characteristic of the entity theory.
c. Being characteristic of the proprietary theory.
d. Why partnerships are characterized by unlimited liability.
e.
PROB. 1-27 (AICPA)

The ABC Partnership reports net income of P60,000. If Partners A, B, and C have
income ratio of 50%, 30%, and 20%, respectively. What is the share of Partner C
from the net income of the partnership, if he was given a capital ratio of 25%?

a. 30,000
b. 12,000
c. 18,000
d. 5,000

PROB. 1-28 (RPCPA)

In the calendar year 2016, the partnership of A and B realized a net profit of
P240,000. The capital accounts of the partners show the following postings:

A, capital B, capital
Debit Credit Debit Credit
Jan 1 P 120,000 P 80,000
May 1 P 20,000 P 10,000
July 1 20,000
Aug 1 10,000
Oct 1 10,000 5,000

If the profits are to be divided based on average capital, the share of A and B,
respectively are:

a. 129,600 110,400
b. 144,000 96,000
c. 136,800 103,200
d. 136,543 103,457

a. 121,500 118,500
b. 124,000 116,000
c. 123,000 117,000
d. 122,625 117,375 If 20% interest
based on capital at the end of the year is allowed and given and the balance of the
240,000 profit is divided equally, the total share of A and B, respectively are:

PROB. 1-29 (AICPA)

During 2016, Young and Zinc maintained average capital balances in their
partnership of P 160,000 and P 100,000, respectively. The partners receive 10%
interest on average capital balances, and residual profit or loss is divided equally.
Partnership profit before interest was P4,000. By what amount should Zinc's capital
account change for the year?

a. 1,000 decrease
b. 2,000 increase
c. 11,000 decrease
d. 12,000 increase

PROB. 1-30 (AICPA)


Red and White formed a partnership in 2016. The partnership agreement provides for
annual salary allowances ofP55,000 for Red and P45,000 for White. The partners
share profits equally and losses in a 60/40 ratio. The partnership had earnings of
P80,000 for 2016 before any allowance to partners. What amount of these earnings
should be credited to each partner's capital account?

Red White
a. 40,000 40,000
b. 43,000 37,000
c. 44,000 36,000
d. 45,000 35,000

PROB 1-31 (AICPA)

Fox, Greg, and Howe are partners with average capital balances during 2016 of P
120,000, P60,000, and P40,000, respectively. Partners receive 10% interest on their
average capital balances. After deducting salaries of P30,000 to Fox and P20,000 to
Howe, the residual profit and loss is divided equally. In 2016, the partnership
sustained a P33,000 loss before interest and salaries to partners. By what amount
should Fox's capital account change?

a. 7,000 increase
b. 11,000 decrease,
c. 35,000 decrease
d. 42,000 increase
PROB. 1-32 (Adapted)
If a partnership has net income of P44,000 and Partner X is to be allocated bonus of
of income after the bonus. What is the amount of bonus Partner X will receive?
a. 3,000
b. 3,300
c. 4,000
d. 4,400

PROB. 1-33 (AICPA)

The partnership agreement of Donn, Eddy, and Farr provides for annual distribution
of profit and loss in the following sequence:

 Donn, the managing partner, receives a bonus of 10% of profit.


 Each partner receives 6% interest on average capital investment.
 Residual profit or loss is divided equally.

Average capital investments for 2016 were:


Donn P80,000
Eddy 50,000
Farr 30,000

What portion of the P 100,000 partnership profit for 2016 should be allocated to Farr?

a. 28,600
b. 29,800
c. 35,133
d. 41,600

PROB. 1-34 (Adapted)

The Articles of Partnership of Adam and Eve the following provisions were stipulated:

a. Annual salary of P60,000 each.


b. Bonus to Adam of 20% of the net income after partners' salaries, the bonus
being treated as an expense.
c. Balance to be divided equally.

The partnership reported a net income of P360,000 after partners' salaries but before
bonus. How much is the share of Eve in the profit?

a. 60,000
b. 90,000
c. 150,000
d. 210,000

PROB. 1-35 (Adapted)


Partners AA and BB have profit and loss agreement with the following provisions:
salaries of P30,000 and P45,000 for AA and BB, respectively; bonus to AA of 10% of
net income after salaries and bonus; and interest of on average capital balances of
P20,000 and P35,000 for AA and BB, respectively. One-third of any remaining profits
will be allocated to AA and the balance to BB. If the partnership had net income of P
102,500, how much should be allocated to Partner AA?

a. 44,250
b. 47,500
c. 41,000
d. 41,167

PROB. 1-36 (Adapted)

Partners AA and BB have profit and loss agreement with the following provisions:
salaries of P30,000 and P45,000 for AA and BB, respectively; a bonus to AA of 10%
of net income after salaries and bonus; and interest of 10% on average capital
balances of P20,000 and P35,000 for AA and respectively. One-third of any
remaining profits will be allocated to AA and the balance to BB. If the partnership had
net income of P22,000, how much should be allocated to Partner AA, assuming that
the provisions of the profit and .10ss agreement are ranked by order of priority
starting with salaries?

a. 13,200
b. 12,500
c. 12,000
d. 8,800

PROB. 1-37 (Adapted)

Luz, Vi, and Minda are partners when the partnership earned a profit of P30,000.
Their agreement provides the following regarding the allocation of profits and losses:

a. 8% interest on partners' ending capital in excess of P75,000.


b. Salaries of P20,000 for Luz and P30,000 for Vi.
c. Any balance is to be distributed 2:1:1 for Luz, Vi, and Minda, respectively.

Assume ending capital balances of P60,000, P80,000, and P 100,000 for partners
Luz, Vi, and Minda, respectively. What is the amount of profit allocated for Minda, if
each provision of the profit and loss agreement is satisfied to whatever extent
possible using the priority order shown above?

a. (3,600)
b. 3,600
c. (2,000)
d. 2,000

PROB. 1-38 (Adapted)

XYZ Partnership provided for the following in their distribution of profits and losses:
First: X to receive 10% of net income up to P 100,000 and 20% of the amount in
excess thereof.
Then: Y and Z are each to receive 5% of the remaining income in excess of PI 50,000
after X's share.
Finally: The balance is to be distributed equally to the three partners.

If the partnership earned a net income of P250,000, what is the total share of Partner
X?

a. 100,000
b. 108,000
c. 110,000
d. 130,000

PROB 1-39 (AICPA)

Hanz, Ivy, Jasper, and Kelly own a publishing company that they operate as a
partnership. Their agreement includes the following:

 Hanz will receive a salary of P20,000 and a bonus of 3% of income after all the
bonuses.
 Ivy will receive a salary of P10,000 and a bonus of 2% of income after all the
bonuses.
 All partners are to receive the following: Hanz- P5,000; Ivy-P4,500; Jasper-
P2,000; and Kelly –P4,700, representing 10% interest on their capital
balances.
 Any remaining profits are to be divided equally among the partners.

a. How would net loss of P40,000 would be allocated among the partners?
Hanz Ivy Jasper Kelly
a. 3,261.75 (7,169.25) (18,181.25) (17,911.25)
b. 3,450.00 (7,050.00) (19,550.00) (16,850.00)
c. 4,116.75 (6,764.25) (20,026.25) (17,326.25)
d. 45,000.00 4,500.00 (8,000.00) (5,300.00)

b. Assuming a profit of P40,000, how would this amount be distributed to them


given the following order of priority: Interest on invested capital, then bonuses,
then salary, and then according to profit and loss percentage?

Hanz Ivy Jasper Kelly


a. 23,261.75 12,830.75 1,818.75 2,088.75
b. 20,867.00 12,433.00 2,000.00 4,700.00
c. 20,740.00 12,560.00 2,000.00 4,700.00
d. 18,038.00 12,562.00 2,000.00 4,700.00

PROB. 1-40 (Adapted)

On October 31, 2016, Zita and Jones formed a partnership by investing cash of
P300,000 and 200,000 and respectively. The partners agreed to receive an annual
salary allowance of P360,000, and to give Zita a bonus of 20% of the net income after
partners’ salaries, the bonus being treated as an expense. If the profits after salaries
and bonus are to be divided equally, and the profits on December 31,2016 after
partners' salaries but before bonus of Zita is P360,000, how much is the share of Zita
in the profit?

a. 100,000
b. 120,000
c. 210,000
d. 270,000

PROB. 1-41 (AICPA)

Maxwell is trying to decide whether to accept a salary of P40,000 or salary of P25,000


plus a bonus of 10% of net income after salaries and bonus as a means of allocating
profit among partners. Salaries traceable to the other partners are estimated to be P
100,000. What amount of income would be necessary so that Maxwell would consider
choices to be equal?

a. 165,000
b. 290,000
c. 265,000
d. 305,000

PROB. 1-42 (AICPA)

A partnership has the following accounting amounts:

Sales P 700,000
Cost of goods sold 400,000
Operating expenses 100,000
Salary allocations to partners 130,000
20,000
Interest paid to banks 80,000
Partners' drawings

What is the partnership net income (loss)?


a. 200,000
b. 180,000
c. 50,000
d. (30,000)

PROB. 1-43 (Adapted)

Alder, Benson, and Carl are capitalist partners and Denver, an industrial partner. The
partnership reported a net loss of P100,000. How much is the share of Denver in the
reported net loss?

a. 0
b. 10,000
c. 25,000
d. 100,000

PROB. 1-44 (Adapted)


If a new partner acquires a partnership interest directly from the partners rather than
from the partnership itself,

a. No entry is required.
b. The partnership assets should be revalued.
c. The existing partners' capital accounts should be reduced and the new
partner's account increased.
d. The partnership has undergone a quasi-reorganization.

PROB. 1-45 (AICPA)

Which of the following results in dissolution of a partnership?

a. The contribution of additional assets to the partnership by an existing partner.


b. The receipt of a draw by an existing partner.
c. The winding up of the partnership and the distribution of remaining assets to
the partners.
d. The withdrawal of a partner from a partnership

PROB. 1-46 (AICPA)

When a new partner is admitted to a partnership, an original partner’s capital account


may be adjusted for

a. A proportionate share of the incoming partner’s investment.


b. His or her share of previously unrecorded intangible assets traceable to the
original partners.
c. His or her share of previously unrecorded intangible assets to the incoming
partner.
d. None of the above.

PROB. 1-47 (AICPA)

Which of the following best characterizes the bonus method of recording a new
partner's investment in a partnership?

a. Net assets of the previous partnership are not revalued.


b. The new partner's initial capital balance is equal to his or her investment.
c. Assuming that recorded assets are properly valued, the book value of the new
partnership is equal to the book value of the previous partnership and the
investment of the new partner.
d. The bonus always results in an increase to the previous partners' capital
balances.

PROB. 1-48 (AICPA)

If goodwill is traceable to the previous partners, it is

a. Allocated among the previous partners according to their interest in capital.


b. Allocated among the previous partners only if there are not other assets to be
revalued.
c. Allocated among the previous partners according to their original profit and
loss sharing percentages.
d. Not possible for goodwill to also be traceable to the incoming partner.

PROB. 1-49 (Adapted)

The goodwill and the bonus methods are two means of adjusting for differences
between the net book value and the fair market value of partnership when new
partners are admitted. Which of the following statements about these methods is
correct?

a. The bonus method does not revalue assets to market values.


b. The bonus method revalues assets to market values.
c. Both methods result in the same balances in the partner capital accounts.
d. Both methods result in the same total value of partner capital account, but the
individual capital account vary.

PROB. 1-50 (AICPA)

Blau and Rubi are partners who share profits and losses in the ratio of 6:4,
respectively. On May l, 2016, their respective capital accounts were as follows:

Blau 60,000
Rubi 50,000

On that date, Lind was admitted as a partner with one-third interest in capital, and
profits for an investment of P40,000. The new partnership began with total capital of P
150,000. Immediately after Lind's admission, Blau's capital should be

a. 50,000
b. 54,000
c. 56,667
d. 60,000

PROB. 1-51 (AICPA)

Partnership A has an existing capital of P70,000. Two partner currently own the
partnership and split profits 50/50. A new partner is to be admitted and will contribute
net assets with a fair value of P90,000. For no goodwill or bonus (depending on
whichever method is used) to be recognized, what is the interest in the partnership
granted the new partner?

a. 33.33%
b. 50.00%
c. 56.25%
d. 75.00%

PROB. 1- 52 (AICPA)

Dunn and Grey are partners with capital account balances of P60,000 and P90,000,
respectively. They agree to admit Zorn as a partner with one-third interest in capital
and profits, for an investment of P 100,000, after revaluing the assets of Dunn and
Grey. Goodwill to the original partner should be

a. 0
b. 33,333
c. 50,000
d. 66,667

PROB. 1-53 (RPCPA)

Mitz, Marc, and Mart are partners sharing profits in the ratio of 5:3:2, respectively. As
of December 31, 2016, their capital balances were P95,000 for Mitz, P80,000 for
Marc, and P60,000 for Mart. On January l, 2017, the partners admitted Vince as a
new partner and according to their agreement, Vince will contribute P80,000 in cash
to the partnership and also pay PIO,000 for 15% of Marc's share. Vince will be given
a 20% share in profits, while the original partner share will be proportionately the
same as before. After the admission of Vince, the total capital will be P330,000 and
Vince's capital will be P70,000.

a. The total amount of goodwill to the old partners, upon the admission of Vince
would be:

a. 7,000
b. 15,000
c. 22,000
d. 37,000

b. The balance of Marc's capital, after the admission of Vince would be:

a. 72,600
b. 74,600
c. 79,100
d. 81,100

PROB. 1 - 54 (AICPA)

Ranken purchases 50% of Lark's capital interest in the K and L partnership for
P22,000. If the capital balances of Kim and Lark are P40,000 and P30,000,
respectively, Ranken's capital balance following the purchase is
a. 22,000
b. 35,000
c. 20,000
d. 15,000

PROB. 1- 55 (Adapted)

The following information pertains to ABC Partnership of Amor, Bing, and Cora:
Amor, capital (20%) P 200,000
Bing, capital (30%) 200,000
Cora, capital (50%) 300,000
On this date, the partners agreed to admit Dolly into the partnership. Assuming Dolly
purchased fifty percent of the partners capital and pays P500,000 to the old partners,
how would this amount be distributed to them?
a. 100,000 150,000 250,000
b. 130,000 145,000 225,000
c. 166,667 166,667 166,666
d. 150,000 150,000 200,000

PROB. 1-56 (AICPA)

The following balance sheet is presented for the partnership of A, B, and C, who
share profits and losses in the respectively ratio of 5:3:2.
Assets Liabilities and Capital
Cash 120,000 Liabilities 280,000
Other assets A, capital 560,000
B, capital 320,000
C, capital 40,000
Total 1,200,000 Total 1,200,000

Assume that the assets and liabilities are fairly valued on the balance sheet, and the
partnership decided to admit D as a new partner with a one-fifth interest and no
goodwill or bonus is to be recorded. How much should D contribute in cash or other
assets?

a. 147,200
b. 184,000
c. 230,000
d. 240,000

PROB. 1-57 (RPCPA)

A, B, and C are partners, who share profits and losses in the ratio of 5:3:2,
respectively. They agree to sell D 25% of their respective capital and profits and
losses ratio for a total payment directly to the partners in the amount of
P140,000.They agree that goodwill of P60,000 is to be recorded prior to the
admission of D The condensed balance sheet of the ABC Partnership is as follows:

Cash P 60,000 Liabilities P 100,000


Noncash assets 540,000 A, capital 250,000
B, capital 150,000
C, capital 100,000
Total P600,000 Total P600,000

The capitals of A, B, and C, respectively after payment and admission of D are:

a. 187,500 112,500 75,000


b. 210,000 126,000 84,000
c. 280,000 168,000 112,000
d. 250,000 150,000 100,000
PROB. 1 - 58 (Adapted)

Fernando and Jose are partners with capital balances of P30,000 and
P70,000, respectively. Fernando has a 30% interest in profits and losses. All
assets of the partnership are at fair market value except equipment with book
value of P300,000 and fair market value of P320,000. At this time, the
partnership has decided to admit Rosa and Linda as new partners. Rosa
contributes cash of P55,000 for a 20% interest in capital and a 30% interest
in profits and losses. Linda contributes cash of P 10,000 and an equipment
with a fair market value of P50,000 for a 25% interest in capital and a 35%
interest in profits and losses. Linda is also bringing special expertise and
clients contact into the new partnership.

a. Using the bonus method, what is the amount of bonus?


a. 24,750
b. 18,250
c. 14,000
d. 7,500

b. Using the goodwill method, what is the amount of goodwill traceable to the
original partners?
a. 60,000
b. 40,000
c. 31,250
d. 28,750

PROB. 1-59 (Adapted)

The capital balances in DEA Partnership are: D, capital P60,000; E, capital P50,000;
and A, capital P40,000 and income ratios are: 5:3:2, respectively. The DEAR
Partnership is formed by admitting R to the firm with cash investment of P60,000 for a
25% interest in capital. What is the amount of bonus to be credited to A capital in
admitting R?

a. 10,000
b. 7,500
c. 3,750
d. 1,500

PROB. 1-60 (AICPA)

Assets, net of liabilities P320,000


Eddy, capital (50%) P160,000
On Fox, capital (30%) 96,000 June 30,
2016, Grimm, capital (20%) 64,000 the
P320,000 condensed
balance sheet for
the
partnership of Eddy, Fox, and Grimm together with their respective profit and loss
sharing percentage, was as follows:

a. Eddy decided to retire from the partnership and by mutual ageement is to be


paid P180,000 out of partnership funds for his interest. Total goodwill implicit in
the agreement is to be recorded. After Eddy's retirement, what are the capital
balances of the other partners?

Fox Grimm
a. 84,000 56,000
b. 102,000 68,000
c. 108,000 72,000
d. 120,000 80,000

b. Assume instead that Eddy remains in the partnership and that Hamm is
admitted as a new partner with a 25% interest in the capital of the new
partnership for a cash payment of P 140,000. Total goodwill implicit in the
transaction is to be recorded. Immediately after admission of Hamm, Eddy's
capital account balance should be

a. 280,000
b. 210,000
c. 160,000
d. 140,000

PROB. 1-61 (Adapted)

In May 2016, Imelda, a partner of an accounting firm, decided to withdraw when the
partners' capital balances were: Mikee, P600,000; Raul, P600,000; and Imelda,
P400,000. It was agreed that Imelda is to take the partnership's fully depreciated
computer with a second hand value of P24,000 that cost the partnership P36,000. If
profits and losses are shared equally, what would be the capital balances of the
remaining partners after the retirement of Imelda?

Mikee Raul
a. 600,000 600,000
b. 592,000 592,000
c. 608,000 608,000
d. 612,000 612,000

PROB. 1-62 (AICPA)

The following condensed balance sheet is presented for the partnership of Alfa and
Beda, who share profits and losses in the ratio of 60:40, respectively:

Cash 45,000
Other assets 625,000
Beda, loan 30,000
700,000
a. The assets and liabilities are fairly
valued on the Accounts payable 120,000 balance sheet. Alfa
and Beda Alfa, capital 348,000 decide to admit Capp
as a new Beda, capital 232,000 partner with a 20%
interest. No 700,000 goodwill or bonus is to
be recorded. What amount should Capp contribute in cash or other assets?
a. 110,000
b. 116,000
c. 140,000
d. 145,000

b. Instead of admitting a new partner, Alfa and Beda decide to liquidate the
partnership. If the other assets are sold for P500,000, what amount of the
available cash should be distributed to Alfa?

a. 255,000
b. 273,000
c. 327,000
d. 348,000

PROB. 1-63 (Adapted)

Penny, Naty, and Mary are partners and share profits and losses equally. Each has a
capital balancer of P1,800,000. Naty retires from the partnership and receives
P1,500,000 Taking the partnership assets to be fairly stated, the entry to record
Naty's retirement is

a. Naty, capital 1,800,000 (dr)


Goodwill 300,000 (cr)
Cash 1,500,000 (cr)

b. Naty, capital 1,800,000 (dr)


Partnership assets 300,000 (cr)
Cash 1,500,000 (cr)

c. Naty, capital 1,500,000 (dr)


Cash 1,500,000 (cr)

d. Naty, capital 1,800,000 (dr)


Mary, capital 150,000 (cr)
Penny, capital 150,000 (cr)
Cash 1,500,000 (cr)

PROB. 1-64 (AICPA)

On June 30, 2016, the balance sheet for the partnership of Coll, Maduro, and Prieto,
together with their respective profit and loss ratios, were as follows:

Assets, at cost 180,000

Coll, loan 9,000


Coll, capital (20%) 42,000
Maduro, capital (20%) 39,000
Prieto, capital (60%) 90,000
Total 180,000
Coll decided to retire from the partnership. By mutual agreement, the assets are to be
adjusted to their fair value ofP216,000 at June 30, 2016. It was agreed that the
partnership would pay Coll P61,200 cash for Coll's partnership interest, including
Coll's loan which is to be repaid in full. No goodwill is to be recorded.
After Coll's retirement, what is the balance of Maduro's capital account?

a. 36,450
b. 39,000
c. 45,450
d. 46,200

PROB. 1- 65 (Adapted)

On October 31, 2016, Morris retired from the partnership of Morris, Philip, and Marl.
Morris received P55,000 representing final settlement of his interest in the amount
ofP50,000. Under the bonus method,

a. P5,000 was recorded as goodwill.


b. P5,000 was recorded as expense.
c. Charged P5,000 against the capital balances of Philip and Marl.
d. P55,000 was recorded as bonus.

PROB. 1 - 66 (Adapted)
Peter, Queen, and Roy are partners with capital balances of P300,000, P300,000,
and P200,000, respectively; and sharing profits and losses equally. Roy is to retire
and it is agreed that he is to take certain office equipment with second hand value of
P50,000 and a note for his interest. The office equipment carried in the books at
P65,000 but brand new would cost Roy's acquisition of the office equipment would
result in

a. Reduction in capital of P5,000 each for Peter, Queen, and Roy.


b. Reduction in capital of P7,5000 each for Peter, Queen, and Roy.
c. Reduction in capital of P15,000 for Roy.
d. Reduction in capital of P55,000 for Roy.

PROB. 1-67 (RPCPA)

N, X, and Y are sharing profits and losses in the ratio of 4:3:3 respectively.
The condensed balance sheet of NXY Partnership as of December 31, 2016
is:

Cash P 50,000 Liabilities P 40,000


Other assets 130,000 N, capital 60,000
X, capital 40,000
Y, capital 40,000
Total P 180,000 Total P 180,000

a. All the partners agree to admit Z as a 1/5 partner in the partnership without
any goodwill or bonus. Z shall contribute assets amounting to

a. 28,000
b. 10,000
c. 35,000
d. 60,000
b. The NXY Partnership is dissolved and liquidated by installments. The first
realization of P40,000 cash is on the sale of other assets with book value of
P80,000. After payment of the liabilities, the cash available is distributed to
N, X, and Y, respectively as follows:

a. 36,000 27,000 27,000


b. 44,000 28,000 28,000
c. 16,000 12,000 12,000
d. 24,000 13,000 13,000

PROB. 1-68 (AICPA)


Gerber, Williams, and George are partners with present capital balances of
P50,000, P60,000, and P20,000, respectively. The partners share profit and
losses according to the following percentages: 60% for Gerber, 20% for
Williams, and 20% for George. Larsen is to joint the partnership upon
contributing P60,000 to the partnership in exchange for a 25% interest in capital
and a 20% interest in profits and losses. The existing assets of the original
partnership are undervalued by P22,000. The original partners will share the
balance of profits and losses in proportion to their original percentages. What
would be the capital balances of the old partners in the new partnership using
the goodwill method?

Gerber Williams George


a. 63,200 64,400 24,400
b. 93,200 74,400 34,400
c. 76,800 65,600 25,600
d. 80,000 70,000 30,000

PROB. 1-69 (AICPA)

The following is the priority sequence in which liquidation proceeds will be distributed
for a partnership:

a. Partnership drawings, partnership liabilities, partnership loans, partnership


capital balances
b. Partnership liabilities, partnership loans, partnership capital balances.
c. Partnership liabilities, partnership loans, partnership drawings, partnership
capital balances.
d. Partnership liabilities, partnership capital balances, partnership loans.

PROB. 1-70 (Adapted)

In accounting for the lump-sum liquidation of a partnership, cash payments to partners


after all non-partner creditors' claims have been satisfied, but before the final cash
distribution, should be according to

a. The partners' relative profit and loss sharing ratio.


b. The final balances in partner capital accounts.
c. The partners' relative share of the gain or loss on liquidation.
d. Safe payment computations.

PROB. 1-71 (Adapted)

In a partnership liquidation, the final cash payment to the partners should be made in
accordance with the

a. Partner's profit and loss sharing ratio.


b. Balances of partners' capital accounts.
c. Ratio of the capital contributions by partners.
d. Safe payment computations.
PROB. 1-72 (AICPA)

The doctrine of marshaling of assets

a. Is applicable only if the partnership is insolvent.


b. Allows partners to first contribute personal assets to unsatisfied partnership
creditors.
c. Is applicable if either the partnership is insolvent or individual partners are
insolvent.
d. Amount owed to personal creditors and to the partnership for debit capital
balances are shared proportionately from the personal assets of the partners.

PROB. 1-73 (AICPA)

Cohen, Butler, and Davis are partners in a partnership and share profits and losses
50%, 30%, and 20%, respectively. The partners have agreed to liquidate the
partnership and anticipate that liquidation expenses will total P14,000. Prior to the
liquidation, the partnership balance sheet reflects the following book

Cash 21,000
Non-cash assets 248,000
Notes payable to Davis 32,000
Other liabilities 154,000
Cohen, capital 60,000
Butler, capital (deficit) (10,000)
Davis, capital 33,000

Assuming that the actual liquidation expenses are P 14,000 and that non-cash assets
are sold for P218,000, how would the assets be distributed to partners if Butler has
net personal assets ofP8,500?

Cohen Butler Davis


a. 15,500 - -
b. 21,429 - 49,571
c. 30,650 - 53,260
d. 27,500 - 52,000

PROB. 1-74 (AICPA)

The following condensed balance sheet is presented for the partnership of Axel, Barr,
and Cain, who share profits and losses in the ratio of 4:3:3, respectively:

Cash P100,000
Other assets 300,000
Total P400,000
Liabilities P150,000
Axel, capital 40,000
Barr, capital 180,000
Cain, capital 30,000
Total P400,000

The partners agreed to dissolve the partnership after selling the other asset for
P200,000. Upon dissolution of the partnership, Axel should have received

a. 0
b. 40,000
c. 60,000
d. 70,000

PROB. 1-75 (Adapted)

Because of very unprofitable operations, partners Nal, Lou, and Gee decided to
dissolve the partnership when their capital balances and profit and loss ratio were:

Nal, capital (30%) P175,000


Lou, capital (20%) 125,000
Gee, capital (50%) 175,000
Total P475,000

Upon liquidation, all of the partnership's assets are sold and sufficient cash is realized
to pay all liabilities except one for P2S,000. Gee is personally insolvent but the others
are capable of meeting any indebtedness of the firms By what amount would the
capital of Nal change?

a. 7,500 decrease
b. 150,000 decrease
c. 195,000 decrease
d. No change

PROB. 1-76 (RPCPA)

Peter and John, who share profits and losses equally, decided to liquidate their
partnership when their net assets amounted to P260,000, and capital balances of PI
70,000 and P90,000, respectively. If the noncash assets were sold for amount equal
to its book value, what amount of cash should Peter and John received?

Peter John
a. 130,000 130,000
b. 170,000 90,000
c. 180,000 80,000
d. 195,000 65,000

PROB. 1-77 (Adapted)

Sammy and Michael are partners of SM Partnership sharing profits and losses
equally. They decided to terminate the partnership when their capital balances are:
Sammy, P750,000; Michael, P500,000. At this time, the partlership owes Michael
P200,000, as evidenced by a promissory note. Upon liquidation, cash of P300,000
becomes available for distribution to the partners. In the final cash distribution, what
would be the respective share of Sammy and Michael?
Sammy Michael
a. 150,000 150,000
b. 175,000 125,000
c. 200,000 100,000
d. 275,000 25,000

PROB. 1-78 (AICPA)

The following condensed balance sheet is presented for the partnership of Smith and
Jones, who share profits and losses in the ratio of 60:40, respectively:
Other assets P 450,000

Smith, loan 20,000


P470,000

Accounts payable 120,000


Smith, capital 195,000
Jones, capital 155,000
P470,000

The partners decided to liquidate the partnership, If the other assets are sold for
P385,000, what amount of the available cash should be distributed to Smith?
a. 136,000
b. 156,000
c. 159,000
d. 195,000

PROB. 1-79 (RPCPA)

The condensed balance sheet of Alex, Jay, and John as of March 31, 2016 follows:

Cash P 28,000 Liabilities P 48,000


Other assets 265,000 Alex, capital 95,000
Jay, capital 80,000
Total P293,000 Total P293,000

The income and loss ratio is 50:25:25, respectively. The partners voted to dissolve
their partnership and liquidate by selling other assets in installments. P70,000 was
realized on the first cash sale of other assets with a book value of P150,000. After
settlement with creditors, all cash available was distributed to the partners. How much
cash was received by John?
a. 10,500
b. 20,000
c. 21,250
d. 32,500
PROB. 1 - 80 (Adapted)

On December 31, 2016, the partners of MNP Partnership decided to liquidate their
business. Immediately before liquidation, the following condensed balance sheet was
prepared:

Cash P 50,000 Liabilities 370,000


Noncash assets 900,000 Nieva, loan 80,000
Perez, loan 25.000
Munoz, capital (50%) 312,500
Nieva, capital (30%) 107,500
Perez, capital (20%) 50,000
Total P950,000 Total P950,000

The noncash assets were sold for P400,000. Assuming Perez is the only solvent
partners, what amount of additional cash will be invested by Perez? (rounded to the
nearest peso)

a. 37,143
b. 25,000
c. 5,000
d. 0

PROB. 1-81 (Adapted)

After incurring losses resulting from very unprofitable operations, the Goh Kong Wei
Partnership decided to liquidate when the partners' capital balances were:

Goh, capital (40%) P80,000


Kong, capital (40%) 130,000
Wei, capital (20%) 96,000

The non-cash assets were sold in installment. Available cash were distributed to
partners in every sale of non-cash assets. After the second sale of non-cash assets,
the partners received the same amount of cash in the distribution. And from the third
sale of non-cash assets, cash available for distribution amounts to P28,000, and
unsold non-cash assets has a book value of P 12,500. Using cash priority program,
what amount did Wei received in the third installment of cash?

a. 11,600
b. 8,000
c. 5,600
d. 0

PROB. 1-82 (AICPA)

Partners Able, Baker, and Chapman, who share profit and loss equally, have the
following personal assets, personal liabilities, and partnership capital balances:
Able Baker Chapman
Personal assets P 30,000 P 80,000 P 60,000
Personal liabilities 25,000 50,000 72,000
Capital balances 50,000 (32,000) 70,000

After applying the doctrine of marshalling of assets, the capital balances of Able,
Baker, and Chapman, respectively, would be

a. 50,000 (2,000) 58,000


b. 48,000 0 58,000
c. 49,000 0 57,000
d. 34,000 0 54,000

PROB. 1-83 (Adapted)

Partner Morgan is personally insolvent, owing P600,000. Personal assets will only
bring P200,000 when liquidated. At the same time, Morgan has a credit capital
balance in the partnership of P 120,000. The capital amounts of the other partners
total a credit balance of P250,000. Under the doctrine of marshalling of assets, how
much the personal creditors of Morgan can collect?

a. 120,000
b. 200,000
c. 320,000
d. 570,000

PROB. 1-84 (RPCPA)

As of December 31, the books of AME Partnership showed capital balances of A -


P40,000; M - P25,000; and E - P5,000. The partners' profit and loss ratio was 3:2:1,
respectively. The partners decided to dissolve and liquidate. They sold all the non-
cash assets for P37,000 cash. After settlement of all liabilities amounting to P 12,000,
they still have P28,000 cash left for distribution.

a. The loss on the realization of the non-cash assets was

a. 40,000
b. 42,000
c. 44,000
d. 45,000

b. Assuming that any partner's capital debit balance is uncollectible, the share of
A in the P28,000 cash for distribution would be
a. 19,000
b. 18,000
c. 17,800
d. 40,000

PROB. 1- 85 (RPCPA)

The balance sheet of the partnership of Salve, Galo, and Norma, who share in the
profits and losses in the ratio of 5:3:2, respectively is as follows:
Assets Liabilities and Capital
Cash 30,000 Liabilities 50,000
Other assets 320,000 Salve, capital 80,000
Galo, capital 115,000
Norma, capital 105,000
Total 350,000 Total 350,000

The partnership is liquidated by installment. The first sale of non-cash assets with a
book value of P 150,000 realizes P 100,000. How should the remaining cash be
distributed?

Salve Galo Norma


a. 50,000 30,000 20,000
b. 40,000 24,000 16,000
c. 0 31,000 49,000
d. 0 48,000 32,000

PROB. 1-86 (RPCPA)

The following balance sheet is presented for the partnership of A, B, and C, who
share profits and losses in the respectively ratio of 5:3:2.

Assets Liabilities and Capital


Cash 120,000 Liabilities 280,000
Other assets A, capital 560,000
B, capital 320,000
C, capital 40,000
Total 1,200,000 Total 1,200,000

Assume that the three partners decided to liquidate the partnership. If the other
assets are sold for P800,000, how should the available cash be distributed to each
partner?

A B C
a. 280,000 320,000 40,000
b. 324,000 236,000 16,000
C. 410,000 230,000 0
d. 412,000 228,000 0

PROB. 1 - 87 (Adapted)
In the liquidation of a partnership it is necessary to (1.) distribute cash to the partners;
(2.) sell non-cash assets; (3.) allocate any gain or loss on realization to the partners;
and (4.) pay liabilities. These steps should be performed in the following order:

a. (2),(3),(4),(1)
b. (2),(3),(1),(4)
c. (3),(2),(1),(4)
d. (3),(2),(4),(1)

PROB. 1-88 (AICPA)

Partners Almond, Barney, and Colors have capital balances of P20,000, P50,000,
and P90,000, respectively. They split profits in the ratio of 2:4:4, respectively. Under a
safe cash distribution plan, one of the partners will get the following total amount in
liquidation before any other partners get anything:

a. 0
b. 15,000
c. 40,000
d. 180,000

PROB. 1 - 89 (AICPA)

The ABC Partnership has assets with book value of P240,000 and a market value of
PI 95,000, outside liabilities of P70,000, loans payable to Partner Able of P20,000,
and capital balances for Partners Able, Baker, and Chapman of P70,000, P30,000,
and P50,000, respectively. The partners share profits and losses equally.

a. How would the first P 100,000 of available assets be distributed?

a. P70,000 to outside liabilities, P20,000 to Able, and the balance equally


among partners.
b. P70,000 to outside liabilities, and P30,000 to Able.
c. P70,000 to outside liabilities, P25,000 to Able, and P5,000 to Chapman.
d. P40,000 to Able, P20,000 to Chapman, and the balance equally among
partners.

b. If all outside creditors and loans to partners had been paid. How would the
balance of the assets be distributed assuming Chapman had already received
assets with a value of P30,000?

a. Each of the partners would received P25,000.


b. Each of the partners would received P40,000.
c. Able: P70,000, Baker: P30,000, Chapman: P20,000
d. Able: P55,000, Baker: P15,000, Chapman: P5,000.

PROB. 1-90 (RPCPA)


Roy Debit Credit and Gil are
Cash P 45,000 partners
Accounts Receivable (net) 60,000 sharing
Inventory 90,000 profits and
Fixed Assets (net) 174,000 losses in the
ratio Liabilities P 60,000 of 1:2,
Roy, Capital 94,800 respectively.
On Gil, Capital 214,200 July l, 2016,
they P 369,000 P369,000 decided to
form the R&G
Corporation by transferring the assets and liabilities of
the partnership to the corporation in exchange for the latter's stock. The following is
the post-closing trial balance of the partnership.

It was agreed that adjustments be made to the following assets to be transferred to


the corporation:

Accounts receivable P40,000


Inventory 68,000
Fixed assets 180,600

The R&G Corporation was authorized to issue P 100 par preferred stock and PIO par
common stock. Roy and Gil agreed to receive for their equity in the partnership 720
shares of the common stock each, plus even multiples of 10 shares of preferred stock
for their remaining interests.

a. The total number of shares of preferred and common stocks issued by


the corporation in exchange for the assets and liabilities of the
partnership are:

Preferred Common
a. 2,540 shares 1,500 shares
b. 2,592 shares 1,440 shares
c .2,642 shares 1,440 shares
d. 2.642 shares 1,550 shares
b. The distribution of the stocks to Roy and Gil would be:

Roy Gil
Preferred Common Preferred Common
785 shares 720 shares 1,384 shares 720 shares
773 shares 750 shares 1,843 shares 750 shares
758 shares 720 shares 1,834 shares 720 shares
738 shares 720 shares 1,758 shares 720 shares

PROB. 1 - 91 (AICPA)

Current Assets P 250,000 The condensed


balance sheet of
Equipment (net) 30,000 Adams & Gray, a
Total assets P 280,000 partnership, at
December 31,
2016, follows:
Liabilities P20,000
Adams, capital 160,000
Gray, capital 100,000
Total liabilities and capital P 280,000

On December 3 1, 2016, the fair values of the assets and liabilities were appraised at
P240,000 and P20,000, respectively, by an independent appraiser. On January 2,
2017, the partnership was incorporated and 1,000 shares of P5 par value common
stock were issued. Immediately after the incorporation, what amount should the new
corporation report as additional paid in capital?

a. 275,000
b. 260,000
c. 215,000
d. 0

SOLUTIONS AND EXPLANATIONS

PROB. 1-1 Suggested answer (a) Limited liability


In a partnership, each partner is personally and individually liable for all partnership
liabilities. In other words, the liability of the partners in a partnership is unlimited.

PROB. 1-2 Suggested answer (c) A partnership is characterized by limited liability

Partnerships have been affected by the proprietary theory, which looks at the entity
through the eyes of the owners. Characteristics of a partnership that emphasizes that
the entity is viewed as the individual owners include the following:

a. Salaries to partners are viewed as distributions of income rather than a


component of income;
b. Unlimited liability of general partners extends beyond the entity to the
individual partners;
c. Income of the partnership is not taxed at the partnership level but rather than,
is included as part of the partners ' individual taxable income;
d. An original partnership is dissolved upon admission or withdrawal of a partner.

PROB. 1-3 Suggested answer (c) A general partner may be a secured creditor of the
limited partnership

A general partner has a voice in management and has unlimited personal liability.
Anyone, including a secured creditor of the limited partnership, may be a general
partner if he/she takes on these responsibilities.

PROB. 1-4 Suggested answer (c) A partnership is created by mere agreements of the
partners

A partnership is easily formed and is relatively free from governmental regulations


and restrictions. Decisions can be made quickly on substantive matters affecting the
firm, whereas in a corporation, formal meetings with the board of directors are often
needed.

PROB 1-5 Suggested answer (a) Fair value at the date of contribution

Where a new legal entity exists, noncash assets are permitted to be recorded at its
fair market value; thus, the capital account should be credited for the current fair
value of the assets at the date of the contribution.

PROB 1-6 Suggested answer (d) Market value, Market value

Non-cash assets contributed into the partnership should be recorded at its current fair
value.

PROB. 1-7 Suggested answer (a)

Partnership capital accounts are similar to corporate paid in capital and retained
earnings; while partnership drawing accounts are similar to corporate dividends
accounts.

PROB. 1-8 Suggested answer: (c) Ces

Al capital 50,000
Ben capital (80,000 - 35,000) 45,000
Ces capital 55,000

At the date of the formation of the partnership, all assets contributed by the partners
are recorded in the books of the partnership at their fair values, and all liabilities
assumed by the partnership are recorded at their present values.

PROB. 1-9 Suggested answer (a) 75,000, 80,000, 85,000

Al Sharif Booba
Cash contribution 50,000 80,000 25,000
Store equipment 25,000
Computer 60,000
Capital balances 75,000 80,000 85.000

Noncash assets contributed by the partners into the partnership should be recorded
at its fair market values. In this case, the fair market value is the cash selling price of
the computer and the second hand value of the store
equipment.

PROB. 1-10 Suggested answer: (a) 592,000 750,000

Atta Boy
Capital balances before adjustments 620,000 800,000
a. Uncollectible accounts receivable (20,000) (40,000)
b. Worthless inventories ( 6,000) ( 7,000)
c. Intangible assets written off ( 2,000) ( 3,000)
Adjusted capital balances 592,000 750,000
When
assets other than cash are invested into the partnership, it is necessary for the
partners to agree upon the value of such assets. The assets are recorded in
accordance with the agreement, and the partners' capital accounts are credited for
the amounts of the respective investments. The effects of the adjustments to the
capital accounts should be in accordance with the accounting equation (Asset =
Liabilities + Capital).

PROB. 1-11 Suggested answer: (b) 190,000

Mary capital before adjustments 264,000


Allowance for doubtful accounts (3% x 120,000) (
3,600)
Merchandise inventory 25,000
Prepaid expenses 3,600
Accrued liabilities ( 4,000)
Mary capital after adjustments 285,000

Total partnership capital (285,000/3/5) 475,000


Multiply by Jane interest 2/5____
Cash to be invested by Jane
190,000

Again, when assets other than cash are invested into the partnership, it is necessary
for the partners to agree upon the value of such assets. The assets are recorded in
accordance with the agreement, and the partners' capital accounts are credited for
the amounts of the respective investments. The effects of the adjustments to the
capital accounts should be in accordance with the accounting equation (Asset =
Liabilities + Capital)

In this case where Jane will have an interest of 2/5, Mary should have an interest of
3/5. Since no goodwill or bonus was mentioned in the problem, the adjusted capital of
Mary represents her 3/5 interest, which will be used as basis to determine the total
partnership capital.

PROB. 1-12 Suggested answer (b) 35,000 75,000

Roberts Smith
Cash P 20,000 P 30,000
Inventory 15,000
Building (net of P10,000 mortgage ) 30,000
Furniture & equipment 15,000
P 35,000 P 75,000

Generally, individual capital accounts should be credited for the fair market value, at
the date of contribution, of the assets contributed by that partner. The partner's capital
credit is based upon the net assets contributed by the particular partner, thus the
liabilities assumed by the partnership reduced the fair market value of the building
invested.
PROB. 1-13 Suggested answer: (a)18,000

Contributed Agreed Increase


Capital Capital Decrease)
Grey 60,000 60,000
Redd 20,000 60,000 40,000
Total 80,000 120,000 40,000
80,000 120,000 40,000
The importance of proper valuation of assets invested by partners cannot be
overemphasized. In order to achieve equity, assets invested by partners should be
reported at their fair market value. Fair value is determined by making reference to
the following: cash transactions of the same or similar assets, quoted market prices,
and independent appraisals.
PROB. 1 -14
a. Suggested answer (d) 60,000
The partnership agreement provides for equal initial capital. Thus under the goodwill
method, the capital credit for Redd should be the same as the contribution of Grey,
thereby increasing the total agreed capital to P120,000, which is P40,000 more than
the total contributed capital (goodwill).
b. Suggested answer (b) 20, 000 bonus to Redd

Contributed Agreed Increase


Capital Capital (Decrease)
Grey 60,000 40,000 (20,000)
Redd 20,000 40,000 20,000
Total 80,000 80,000

The partnership agreement provides for equal initial capital. Thus under the bonus
method, the capital credit for Redd should be the same as the contribution of Grey,
resulting to P20, 000 bonus from Grey to Redd.

PROB. 1-15 Suggested answer: (d) 0


Under the bonus method, assets are not revalued, instead, adjustments are made to
partnership capital accounts; consequently, unidentifiable assets are not recognized.
PROB. 1-16 Suggested answer: (b) 500,000 450,000 550,000

Alex Benjie Cesar


Contributions @ fair value P500,000 P800,OOO P550,000
Less liabilities assumed - 350,000 -_____
Capital balance, 4/30/06 P500,000 P450,000 P550,000

Again, any noncash asset contributed into the partnership should be valued at the fair
value of the noncash asset contributed. Any liabilities assumed by the partnership,
reduces the partners’ capital balance. As a general guideline, what is to be recorded
as a credit to partner’ capital is the fair value of the net assets contributed.

PROB 1-17 Suggested answer (d) 187,500 200,000 212,500

Abel Cain Josuah


Cash contributed P 100,000 P 160,000 P 50,000
Noncash contributed 50,000 120,000
Capital balances beginning 150,000 160,000 170,000
Capital beginning Distribution of
net income:

(150/480 x 120,000) 37,500


(160/480 x 120,000) 40,000
(170/480 x 120,000) 42,500
Capital balances, 12/31/16 P187,500 P200,000 P212,500

Noncash assets contributed by partners to form a partnership should be recorded at


its fair value. Profits and losses are divided in accordance with the agreement of the
partners, normally, the profit and loss ratio. In the absence of any agreement, profits
and losses are divided in accordance with the partners ' contributed capital.
PROB 1-18 Suggested answer (b) Flat Flat

Profit (Flat) Loss (Flat)

Bonus 20% Bonus 0

Balance (2/5 x 80%) 32% P & L (2/5 x 100%) 40%

Total advantage 52% Total advantage 40%

In case of profit Flat has 52%, an advantage, and in case of loss, its only 40%, also
an advantage
PROB. 1-19 Suggested answer (c) In all earnings or loss situation

When agreement provides for salaries without qualification, salary distribution must
be made even though profit is inadequate to cover salaries or there is a loss. In this
case, Partner A will benefit by P6, 000 in all situations, whether there is a profit or
loss.

PROB. 1-20 Suggested answer (c) 9, 000

Vick's share of undistributed losses (30% x 30,000) 9,000

If the partners agree to distribute profits based on profit sharing ratio but
are silent on loss sharing, partnership losses will be divided based on the agreed
profit sharing proportions.

PROB. 1-21 Suggested answer: (b) 15,375

Date Balance Months Unchanged Total


January 1 P140,000 6 P 840,000
July 1 180,000 1 180,000
August 1 165,000 5 825,000
Total 12 P 1,845,000
Weighted average capital (1,845,000/ 12months) P153,750
Interest rate 10%
Interest to be credited to Simm P 15,375

When partners wish to distribute profits in terms of relative investments, the use of
average capitals, which provides for the recognition of capital changes during the
period, normally offers the most equitable method. From the data given above,
partner 's investments were expressed in terms of peso month. Under this method
(peso-day, peso-month), withdrawals and investments made during the first half of
the month should be treated as if they were made on the first day of the month, while
withdrawals and investments made during the later half of the month should be
treated as if they were made on the first day of the following month.

PROB. 1-22 Suggested answer (b) 51, 667

Date Balances Months Total


Unchanged
March 1 P 50,000
3 P 150,000
June 1 70,000 3 210,000
September 1 65,000 4 260,000
Total 12 P 620,000

Annual weighted average capital (P620,000/ 12) P 51,667

The partnership agreement should provide how invested capital is to be determined.


Since each partner's equity is a combination of capital and drawing account balances,
partner's drawings may be offset against their respective capital accounts for
purposes of allocating income based on invested capital. However, the agreement
may also provide that only withdrawals more than a certain limit are to be viewed as
offset against capital balances. Thus, only P5,000 excess of P10,000 limit is viewed
as deduction from the capital balance.

PROB. 1-23 Suggested answer (d) In accordance with capital contribution

The ratio in which partnership profits and losses are divided is known as profit and
loss ratio. Profits and losses are divided in accordance with the agreement of the
partners. In the absence of any agreement, profits and losses are divided in
accordance with the partners ' contributed capital.

PROB. 1-24 Suggested answer (d) lnterest on notes to partners

The division of partnership income should be based on an analysis of th correlation


between capital and labor committed to the firm by individual partners and the income
that subsequently is generated. As a result, profits might be divided in one or more of
the following ways: 1.) according ratio; 2.) according to the capital investments of the
partners; and according to the labor (or service) rendered by the partners. Interest on
notes to partners is a legitimate expense of a partnership

PROB. 1-25 Suggested answer: (a) Interest paid to partners based on the amount of
invested capital

Again, the division of partnership income should be based on an analysis of the


correlation between capital and labor committed to the firm by individual partners and
the income that subsequently is generated and therefore includes interest paid to
partners based on the amount of their invested capital.

PROB. 1-26 Suggested answer: (c) Being characteristics of the proprietary theory

Partnerships have been affected by the proprietary theory, which looks at the entity
through the eyes of the owners. Characteristics of a partnership that emphasizes that
the entity is viewed as the individual owners include the following:

a. Salaries to partners are viewed as distributions of income rather than a component


of income;
b. Unlimited liability of general partners extends beyond the entity to the individual
partners;
c. Income of the partnership is not taxed at the partnership level but rather than, is
included as part of the partner’s individual taxable income;
d. An original partnership is dissolved upon dismissal or withdrawal of a partner.

PROB. 1-28

a. Suggested answer: (d) 136,543 103,457

A, capital:
Date Balances Months Unchanged Total
January 1 P120,000 4 P480,000
May 1 100,000 3 300,000
Aug. 1 110,000 2 220,000
Oct. 1 100,000 3 300,000
Total 12 P1,300,000

Average capital – A = P1,300,000/12= P108,333

B, capital:
Date Balances Months Unchanged Total
January 1 P80,000 4 P320,000
May 1 70,000 2 140,000
July 1 90,000 3 270,000
Oct. 1 85,000 3 255,000
Total 12 P985,000

Average capital – A = P985,000/12= P82,083

A P240,000 x (108,333/190417) P136,543


B 240,000 x (82,083/190,417) 103,457
Total P240,000

Again, under this method (peso-day, peso-month), withdrawals and investments made
under during the first half of the month should be treated as if they were made on the
first day of the month, while withdrawals and investments made during the latter half of
the month should be treated as if they were made on the first day of the following month.

b. Suggested answer: (a) 121,500 118,500

A B Total
Interest on ending capital
(100,000 x 20%) P20,000
(85,000 x 20%) P17,000 P37,000
Balance (equally) 101,500 101,500 203,000
Total P121,500 P118,500 P240,000

The capital contributions to be considered as the basis for the distribution of profits and
losses should be based on the original capital contribution, on the capitals at the
beginning of each period, on the capitals at the end of each period, or on the average
capitals during the period.
PROB. 1-29 Suggested answer: (a) 1,000 decrease

Young Zinc Total


10% interest on ave. capital:
(10% x 160,000) P16,000
(10% x 100,000) P10,000 P26,000
Balance (equally) (11,000) (11,000) (22,000)
Total P5,000 (P1,000) P4,000

The partnership profit before interest was P4,000, however, it resulted to a loss of
P22,000 after interest. Thus, the capital balance of Zinc decreases by P1,000.

PROB. 1-30 Suggested answer: (b) 43,000 37,000

Red White Total


Salary allowances 55,000 45,000 100,000
Loss after allowances (60:40) (12,000) (8,000) (20,000)
Earnings credited to partners 43,000 37,000 80,000

The earnings before allowances of P80,000 is reduced by the salary allowances in total
amount of P100,000 which resulted to a loss after allowances of P20,000, because
credits to partners capital accounts are based on earnings after allowances (e.g.
interest, salary, and bonus). It should be pointed out that per partnership agreement
profits should be shared equally and losses in a 60/40 ratio, thus the loss of P20,000
was shared at 60/40 ratio.

PROB. 1-31 Suggested answer: (a) 7,000 increase

Fox Greg Howe


Total
10% interest on
average capital 12,000 6,000 4,000 22,000
Salaries 30,000. 20,000 50,000
Bal. (equally) (35,000) (35,000) (35,000) (105,000)
Total inc. (dec) 7,000 (29,000) (11,000) (33,000)
Again, when the partnership agreement provides without specification that interest is
to be allowed on investment, interest must be allowed even though operations have
resulted in earnings that are less than the allowable interest or in a loss.

And when agreement also provides for salaries without qualification, salary distribution
must be made even though profit is inadequate to cover salaries or there is a loss.
Interest and salary allowances allocated to partners increase their capital balances as
well as the amount of loss. Accordingly, the amount of loss will reduce the partners’
capital accounts. The resulting loss in the total amount of P105,000 after the interest
and salary allowances was allocated among partners equally based on their
agreement, that profit and loss is divided equally.

PROB. 1-32 Suggested answer: (c) 4,000

Net income before bonus 44,000


Less net income after bonus 40,000
Bonus (10%) 4,000

Note that the provision for bonus is 10% of income AFTER the bonus, thus the bonus
is the difference between the net income before and after the bonus.

PROB. 1-33 Suggested answer: (a) 28,600

Donn Eddy Farr


Total
10% bonus to Donn 10,000 10,000
6% interest on
average capital 4,800 3,000 1,800 9,600
Balance (equally) 26,800 26,800 26,800
80,400
Total 41,800 29,800 28,600
100,000

In some instances, a managing partner is allowed a bonus that is to be based on the


earnings of the business. The bonus is commonly stated as a percentage of profits, but
the agreement should indicate whether the percentage is to be applied to the profit
determined before deduction of the bonus or after deduction of the bonus. Based on
the data provided in this problem, the “10% bonus of the profit” was assumed to be
applied to the profit before deduction of the bonus.

PROB. 1-34 Suggested answer: (d) 210,000

Adam Eve Total


Salaries 60,000 60,000 120,000
Bonus to Adam:
NY before bonus P360,000
-NY after bonus
(360,000/120%) 300,000. 60,000 60,000
Balance (equally) 300,000 150,000 150,000 300,000
Total 270,000 210,000 480,000

It should be pointed out that it was clearly mentioned in the problem that the P360,000
net income is after salaries but before bonus, therefore, the net income before
salaries and bonus should be P480,000 (120,000+360,000).

PROB. 1-35 Suggested answer: (c) 41,000

AA BB Total
Salaries 30,000 45,000 75,000
Bonus (after bonus)
NY before Bonus 27,500
NY after Bonus
(27,500/110%) 25,000 2,500 2,500
10% interest 2,000 3,500 5,500
Balance (1/3:2/3) 6,500 13,000 19,500
Total 41,000 61,500 102,500

One of the alternatives in profit allocations if the net income is not sufficient is to
completely satisfy all provisions of the profit and loss agreement and use the profit and
loss ratios to absorb any deficiency or additional loss caused by such action.

PROB 1-36 Suggested answer (d) 8,800


AA BB Total
Salaries (30:45) 8,800 13,200 22,000

Another alternative in profit allocation if the net income is not sufficient is to satisfy each
of the provision to whatever extent it is possible. In other words, the allocation of
salaries would be satisfied to whatever extent possible before the allocation of interest
is begun. If the provision of the profit and loss agreement are ranked by order of priority
starting with salaries, and the total salaries amounted to P75,000, therefore the net
income of P22,000, which is insufficient, will be distributed between AA and BB based
on the degree of salary claims.

PROB. 1-37 Suggested answer: (d) 2,000

Luz Vi Minda Total

Interest
8% x 80,000-75,000 400
8% x 100,000-75,000 2,000 2,400
Salary (20:30) 11,040 16,560
27,600
Total 11,040 16,960 2,000
30,000

Again, where income is not sufficient or an operating loss exists, two alternatives may
be employed: 1.) all provisions of the profit and loss agreement may be satisfied and
any deficiency will be absorbed using the profit and loss ratio; and 2.) each of the
provision may be satisfied to whatever extent possible. The second alternative, as
applied above, requires that provisions of the profit and loss agreement be ranked by
order of priority.

PROB. 1-38 Suggested answer (b) 108,000

X Y Z Total
10% of P100,000 to X 10,000
10,000
20% of excess to X
(20% x 150,000) 30,000 30,000
5% of remaining in excess
to Y and Z
(5% x 210,000-150,000) 3,000 3,000 6,000
Balance, equally 68,000 68,000 68,000
204,000
Total 108,000 71,000 71,000
250,000

Note that the distribution of profit is based on the agreement of the partners.

PROB. 1-39

a. Suggested answer: (b) 3,450 (7,050) (19,550) (16,850)


Hanz Ivy Jasper
Kelly Total
Salaries 20,000 10,000
30,000
Interest 5,000 4,500 2,000 4,700
16,200
Balance (equally) (21,550) (21,550) (21,550) (21,550)
(86,200)
Total 3,450 (7,050) (19,550) (16,850)
(40,000)

Based on the information provided in the problem, the profit and loss ratios are used to
absorb any deficiency or any additional loss.

b. Suggested answer: (c) 20,750 12,560 2,000 4,700

Hanz Ivy Jasper


Kelly Total
Interest 5,000 4,500 2,000 4,700
16,200
Bonus (3:2) 1,143 762
1,905
Salaries (20:10) 14,597 7,298
21,895
Total 20,740 12,560 2,000
4,700 40,000

Net income before bonuses 40,000


Net income after bonuses (40,000/105%) 38,095
Bonuses 1,905

If each of the provision of the profit and loss agreement are satisfied to whatever
extent it is possible based on the given order of priority, at such provision (salaries)
the remaining amount (21,895) shall be allocated using the degree of the claims.

PROB. 1-40 Suggested answer: (d) 270,000

Zita Jones Total


Salaries (360,000 x 2/12) 60,000 60,000 120,000
Bonus 60,000 60,000
Balance (equally) 150,000 150,000 300,000
Total 270,000 210,000 480,000

Net income before bonus 360,000


Net income after bonus (360,000/120%) 300,000
Bonus 60,000

Note that based on the given information, the net income after salaries is P360,000
(300,000+60,000), which is equivalent to P480,000 net income before salaries, bonus
and distribution of balance using profit and loss ratio (equally).

PROB. 1-41 Suggested answer: (b) 290,000

Amount of bonus needed to equalize (40,000-25,000) 15,000

Net income after bonus and salaries (15,000/10%) 150,000


Multiply by 110%
Net income before bonus and salaries 165,000
Add salaries (100,000+25,000) 125,000
Net income before bonus and salaries 290,000

Since the question being asked is the amount of income necessary to equalize, the
appropriate approach is to determine the amount of the difference between the two
alternatives, which is P15,000 bonus. At this point, the net income could be determined
by working back, as shown above.

PROB. 1-42 Suggested answer: (b) 180,000

Sales 700,000
Less cost of goods sold 400,000
Gross profit 300,000
Less operating expenses 100,000
Operating profit 200,000
Less interest paid to banks 20,000
Net income 180,000

Salaries, like interest on capital investments, are viewed as a means of allocating


income rather than as an expense. The drawing account is a temporary account and is
periodically closed to the partner’s capital account, and has nothing to do with the
computation of net income.

PROB. 1-43 Suggested answer: (a) 0

In case there is an industrial partner, and there is no profit and loss sharing agreement,
an industrial partner shall not be liable for the losses. As to profit, the share of an
industrial partner shall be that which is just and equitable under the circumstances. In
order for an industrial partner be liable for the losses, there should be an expressed
stipulation to that effect.

PROB. 1-44 Suggested answer: (c) The existing partner’s capital should be reduced
and the new partner’s account increased.

When a new partner deals directly with an existing partner or partners rather than with
the partnership entity, the acquisition price is paid to the selling partner/s and not to the
partnership itself. The partnership records the redistribution of capital interests by
transferring all or a portion of the seller’s capital to the new partner's capital account
but does not record the transfer of any asset or consideration.
PROB. 1-45 Suggested answer: (d) Withdrawal of a partner from a partnership.

Dissolution is the change in the relation od the partners caused by any partner ceasing
to be associated in the carrying on as distinguished from the winding up of the business.
Generally, a partnership is dissolved upon the death, withdrawal, admission, or
bankruptcy of an individual partner (owner).

PROB. 1-46 Suggested answer: (b) His or her share of previously unrecorded
intangible assets traceable to the original partner.

Accounting change in the ownership of a partnership is influenced heavily by the legal


concept of dissolution. When there is a change in the ownership structure, the original
partnership is dissolved and most often a new partnership is created. This dissolution
and subsequent creation of a partnership indicate that a new legal entity has been
created and accounting should measure properly the initial contributions of capital
being made to the new partnership.

PROB. 1-47 Suggested answer: (c) Assuming that recorded assets are properly
valued, the book value of the new partnership is equal to the book value of the
previous partnership and the investment of the new partner.

Under the bonus method, total contributed capital of the old and new partnership is
equal to the total agreed capital (total capital of the new partnership).

PROB. 1-48 Suggested answer: (c) Allocated among the previous partners according
to their original profit and loss sharing percentages.

Unrecorded goodwill also may be identifiable. If there are no differences between the
fair market value and book value of recorded assets, the new partner’s willingness to
pay more than the proportionate book value of the new entity indicates that goodwill
existed prior to the new partner’s admission. If this intangible asset could have been
sold prior to the admission of the partner, the realized profit would have been allocated
to the old partners. Therefore, the goodwill is recorded and allocated to the old partners
according to their profit and loss ratio.
PROB. 1-49 Suggested answer: (a) The bonus method does not revalue assets to
market values.

When an incoming partner's contribution is different from that indicated by the book
values of th3 original partnership, the admission of the partner typically is recorded by
either the bonus method or th3 goodwill method. Both methods permit the assumption
that there is unrecorded goodwill to be recognized. However, the use of either method
does not prevent the recognition of differences between the book value and fair market
value of recorded net assets.

The bonus method strictly follows the principle that net assets should be recorded at
historical cost and simply readjusts capital accounts and makes no changes in existing
assets accounts. While, the goodwill method emphasizes the legal significance of a
change in the ownership structure of a partnership and revalues assets to adjust the
total value of partnership capital.

PROB. 1-50 Suggested answer: (b) 54,000

Contributed Capital Agreed Capital Increase


(Decrease)
Old partners P110,000 P100,000 (P10,000)
New partners 40,000 (1/3) 50,000 10,000
Total P150,000 P150,000 -

Blau’s capital before admission of Lind P60,000


Less share in bonus to Lind (10,000 x 60%) 6,000
Blau’s capital after admission of Lind P54,000

When a partnership is in urgent need of additional funds or the partners may desire the
services of a certain individual, a new partner may be admitted with the provision that
(a) part of the capitals of the old partners shall be allowed as a bonus to the new partner,
or (b) goodwill shall be established and credited to the new partner.

When the total contributed capital is equal to the agreed capital, there is bonus. In this
case, the amount by which the interest allowed to the new partner exceeds his
investment may be considered as bonus contributed by the old partners. The bonus is
deducted from the capitals of the old partners based in their original profit and loss ratio.
PROB. 1-51 Suggested answer: (c) 56.25%

Capital contributed by the new partner 90,000


Divide by total contributions (70,000+90,000) 160,000
New Partner's interest 56.25%

Since no goodwill or bonus to b3 recognized, the capital contribution is equal to the


capital credited to partners.

PROB. 1-52 Suggested answer: (c) 50,000

Contributed Capital Agreed Capital Increase


(Decrease)
Old partners P150,000 P200,000 P50,000
New partner 100,000 (1/3) 100,000 -
Total P250,000 P300,000 P50,000

When a partnership has operated with considerable success, the partners may admit
a new partner with the provision that (a) part of the new partner’s investment shall be
allowed as a bonus to the old partners, or (b) partnership goodwill shall be established
and credited to the old partners. When the total agreed capital is more than the
contributed capital, there is goodwill. Since the combined capitals of the old partners
was increased to P200,000, the increase in capital of P50,000 should be recognized
as goodwill and distributed to them using their original profit and loss ratio.

PROB. 1-53

a. Suggested answer: (b) 15,000

Contributed Capital Agreed Capital Increase


(Decrease)
Old partners
[235,000-(15%x 80,000)] 223,000 260,000 37,000
New partner
[80,000+(15%x 80,000)] 92,000 70,000
(22,000)
Total 315,000 330,000 15,000
Again, when the total agreed capital is more than the contributed capital, there is
goodwill and this amount of P15,000 will be distributed to the old partners using their
original profit and loss ratio. However, in the above computations, it should be pointed
out that aside from goodwill, the new partner provides bonus to old partners in the
amount of P22,000 (decrease in the new partner’s capital).

b. Suggested answer: (c) 79,100

Marc’s capital before Vince’s admission 80,000


Interest purchased by Vince (15% x 80,000) (12,000)
Share in goodwill (30% x 15,000) 4,500
Share in bonus (30% x 22,000) 6,600
Marc’s capital after Vince’s admission 79,100

When an incoming partner purchases a portion or all of the interest of one or more of
the original partners, the partnership assets remains unchanged and the related
amount paid for the purchase should not be recorded in the books of the partnership
for this is regarded as a personal transaction between the selling partner/s and the
buyer.

Bonus to old partners is recorded by making transfer from the contributed capital of the
new partner to the old partners’ capital accounts, which gives the new partner a capital
credit less than his actual investment. While, Goodwill to old partners should be
recognized by a debit to Goodwill account, and the resulting credit should be to old
partners’ capital accounts allocated based on their profit and loss ratio.

PROB. 1-54 Suggested answer: (d) 15,000

Ranken’s capital (50% x 30,000) 15,000

When a new partner deals directly with an existing partner or partners rather than with
the partnership entity, the acquisition price is paid to the selling partner/s and not to the
partnership itself. The partnership records the redistribution of capital interests by
transferring all or a portion of the seller’s capital to the new partner’s capital account
but does not record the transfer of any asset or consideration.

PROB. 1-55 Suggested answer: (b) 130,000 145,000 225,000


Amor Bing Cora
Interest purchased (1/2) 100,000 100,000 150,000
Excess payment (150,000) P&L 30,000 45,000
75,000
Total 130,000 145,000 225,000

A new partner may be admitted to the partnership by acquiring all or part of the capital
interest of one or more existing partners in exchange for some consideration. The
partnership records the redistribution of capital interest by transferring all or a portion
of the seller’s capital to the new partner’s capital account but does not record the
transfer of any asset. Any difference between the amounts paid by the new partner,
which is not recorded in the books of the partnership, is allocated to the selling partners
based on their profit or loss ratio.

PROB. 1-56 Suggested answer: (c) 230,000

Total capital of the old partnership (560,000+320,000+40,000) 920,000


Divide by profit and loss (old partnership) 4/5
Total capital of the new partnership 1,150,000
Multiply by profit and loss of D 1/5
Required contribution by D 230,000

If the book value of the original partnership’s net assets approximates fair market
values or no bonus, no goodwill to be recognized, the incoming partner’s contribution
would be expected to be equal to the portion of the equity that the new partner is
acquiring.

PROB. 1-57 Suggested answer: (b) 210,000 126,000 84,000

A B C
Capital balances P250,000 P150,000 P100,000
Share in goodwill 5:3:2 30,000 18,000 12,000
Adjusted capital balances 280,000 168,000 112,000
¼ interest purchased (70,000) (42,000) (28,000)
Capital balances after D P210,000 P126,000 P84,000
Again, when an incoming partner purchases a portion or all of the interest of one or
more of the original partners, this may be regarded as a transfer from the capital
account/s of the seller/s to that of the buyer, and any amount paid for the purchase
should not be recorded in the books of the partnership, therefore, the partnership
assets remained the same. The one-fourth interest purchased by D was based on the
adjusted capitals of the old partners (after goodwill) because of the stipulation in the
problem that goodwill is to be recorded prior to the admission of D.

PROB. 1-58

a. Suggested answer: (b) 18,250

Contributed Capital Agreed Capital Increase


(Decrease)
Old partners P100,000 P118,250 P18,250
New partners 115,000 (45%) 96,750 (18,250)
Total P215,000 P215,000 -

The goodwill and bonus method are two means of adjusting for differences between
the net book value and the fair market value of partnership when new partners are
admitted. The goodwill method revalues assets to market value; while the bonus
method does not revalue assets to market value. Further, under the bonus method, the
total contributed capital is equal to the agreed capital.

b. Suggested answer: (c) 31,250

Contributed Capital Agreed Capital Increase


(Decrease)
Old partners P100,000 P151,250 P51,250
New partners 115,000 (45%) 123,750 8,750
Total P215,000 P275,000 P60,000

Total increase in capital 60,000


Less undervalued equipment (320,000-300,000) 20,000
Balance 40,000
Goodwill to Linda 8,750
Goodwill to original partners 31,250
Again, when there is a difference between the book value and fair market value of the
partnership when new partners are admitted, the goodwill method revalues assets to
market value. Ordinarily, to determine the new capital of the partnership, contributed
capital of the new partner may be divided by his interest in capital. In this case, where
Linda will be provided with goodwill for bringing her expertise and clients contact to the
partnership, the capital of Rosa was used instead, because it serves as concrete basis
with no goodwill involved, in determining the new capital of the partnership. Thus, the
new capital of the partnership is P275,000 (55,000/20%).

PROB. 1-59 Suggested answer: (d) 1,500

Contributed Capital Agreed Capital Increase


(Decrease)
Old partners 150,000 157,500 7,500
New partner 60,000 52,500 25% (7,500)
Total 210,000 210,000 -

Bonus to A (7,500 x 20%) 1,500

Under the bonus method of admitting a new partner into the partnership, the total
contributed capital (including that of the new partner) is equal to the new partnership
capital. Accordingly, any bonus to the old partners shall be allocated using their old
profit and loss ratio.

PROB. 1-60

a. Suggested answer: (c) 108,000 72,000

Goodwill to be paid to Eddy (P180,000-160,000) P20,000


Divide by Eddy’s P&L 50%
Total goodwill P40,000

Fox Grimm
Capital balance before goodwill P96,000 P64,000
Goodwill: (40,000x30%) 12,000
(40,000x20%) 8,000
Capital balance after goodwill P108,000 P72,000
Since the problem identified that total goodwill implicit in the agreement is to be
recorded, the excess of the amount received by Eddy over his capital balance
represents his share in the total goodwill to be recognized. Accordingly, Fox and Grimm
will share in the total goodwill based on their respective profit and loss percentage.

b. Suggested answer: (b) 210,000

Contributed Capital Agreed Capital Increase


(Decrease)
Old partners P320,000 P420,000 P100,000
New partner 140,000 (25%) 140,000 -
Total P460,000 P560,000 P100,000

Eddy’s capital before goodwill P160,000


Share in goodwill (50% x 100,000) 50,000
Eddy’s capital after goodwill P210,000

Again, goodwill is the excess of total agreed capital over the contributed capital. In this
case, the amount of P100,000 represents goodwill to old partners, which will be divided
based on their respective profit and loss ratio.

PROB. 1-61 Suggested answer: (c) 608,000 608,000

Mikee Raul

Capital balance before withdrawal 600,000 600,000


Distribution of gain on realization (24,000/3) 8,000 8,000
Capital balances after withdrawal 608,000 608,000

When a partner withdraws, he may receive an amount equal, more than or less than
his interest. The interest of the withdrawing partner is measured by his capital balances
adjusted by the distribution of profit or loss from operations, and changes in valuation
of all assets and liabilities. Thus, their capital balances will be increased by their
respective share in the realization of noncash asset with a fair value different from its
book value at the date of withdrawal.

PROB. 1-62

a. Suggested answer: (d) 145,000


Total capital [(348,000+232,000)/80%] 725,000
Capp’s interest x 20%
Cash or other assets to be contributed by Capp 145,000

It should be pointed out that the problem clearly state that no bonus or goodwill is to be
recognized, thus the total capital of the old partners was used as the basis in computing
the total capital of the partnership.

b. Suggested answer: (b) 273,000

Alfa Beda
Capital balances 348,000 232,000
Beda, loan (30,000)
Total interest 348,000 202,000
Loss on realization (625,000-500,000) 6:4 (75,000) (50,000)
Cash to be distributed 273,000 152,000

Beda, loan account was appropriately presented as an asset (receivable of the


partnership from Beda), therefore it will reduce the capital balance od Beda. And loss
on realization should be charged to the partners’ capital. In partnership liquidation, cash
is distributed bases on the capital balances of the partners after any adjustments. It
should be pointed out that the total amount of cash to be distributed between partners
equals the total partners’ capital after adjustments.

PROB. 1-63 Suggested answer: (d)

Note that the capital balance of the retiring partner is P1,800,000 and was paid
P1,500,000, this situation will result to the recognition of bonus to the remaining
partners from the retiring partner. Thus the difference of P300,000 was allocated to the
remaining partner based on their profit and loss agreement (equally) by crediting their
respective capital account.

PROB. 1-64 Suggested answer: (c) 45,450

Coll Maduro Prieto


Capital balances before
Retirement of Coll P42,000 P39,000 P90,000
Coll, loan 9,000
Adjustments of assets 2:2:6
(216,000-180,000) 7,200 7200 21,600
Total interest 58,200 46,200 111,600
Less payment to Coll 61,200
Balance (3,000) 46,200 111,600
Bonus to Coll 2:6 3,000 (750) (2,250)
Capital balances after
Retirement of Coll P0 P45,450 P109,350

Again, when partner withdraws from a partnership, adjustment of assets its fair market
should be made. Total interest of the withdrawing partner must be determined and be
compared with the amount paid. Since the problem stated that the withdrawing partner
is selling his interest to the partnership and no goodwill is to be recorded, the resulting
difference between the total interest and the amount paid represents the bonus
provided by the remaining partners to the withdrawing partner.

PROB. 1-65 Suggested answer: (c)

Under the bonus method, the excess of the amount paid by the partnership to the
retiring partner shall be absorbed by the remaining partners based on their existing
profit and loss ratio.

PROB. 1-66 Suggested answer: (d) Reduction in capital of P55,000 for Roy.

Peter Queen Roy

Second hand value taken 50,000


Loss on realization
(65,000-50,000) (equally) 5,000 5,000 5,000
Total reduction in capital 5,000 5,000 55,000

PROB. 1-67

a. Suggested answer: (c) 35,000


Total partnership capital (140,000/ 4/5) 175,000
Multiply by 1/5
Assets to be contributed by Z 35,000

Again, if there is no bonus or goodwill to be recognized, total partnership capital may


be computed using the capital accounts of the old partners as the base, as shown
above.

b. Suggested answer: (d) 24,000 13,000 13,000

N X Y
Capital balances P60,000 P40,000 P40,000
Loss on realization
(80,000-40,000) 4:3:3 (16,000) (12,000) (12,000)
Possible loss
(130,000-80,000) 4:3:3 (20,000) (15,000) (15,000)
Cash distribution P24,000 P13,000 P13,000

In installment liquidation, to insure an equitable distribution of cash to the partners,


considerable care is required. Usually, the statement of liquidation is supported by the
Schedule of Safe Payments, wherein each installment of cash is distributed as if no
more cash is forthcoming. Thus, cash is distributed to a partner only if he has an excess
credit balance in his partnership interest after absorption of his share of the maximum
possible loss that may occur, which consists of the assumed realizable value of the
remaining noncash assets, cash withheld for payment of anticipated liquidation
expenses and unrecorded liabilities that may arise, and any additional loss that may
also accrue to the partners when a debit balance in any of the capital accounts results
from the allocations of possible loss. Should the liquidation extend over a long period
of time, these calculations may become frequent such that it may be desirable to
prepare in advance an installment distribution plan, known as Cash Priority Program.

PROB. 1-68 Suggested answer: (d) 80,000 60,000 20,000 130,000

Gerber Williams George Total

Capital contributions 50,000 60,000 20,000


130,000
Asset revaluation 13,200 4,400 4,400 22,000
Total 63,200 64,400 24,400
152,000
Goodwill to old partners 16,800 5,600 5,600
28,000
Total capital 80,000 70,000 30,000
180,000

Contributed Capital Agreed Capital Increase


(Decrease)
Old partners 152,000 180,000 28,000
New partner 60,000 60,000 25% -
Total 212,000 240,000 28,000

Again, under the goodwill method total contributed capital is less than the total agreed
capital.

PROB. 1-69 Suggested answer: (b)

In the event of liquidation, subject to any agreement to the contrary, the following
sequence of payments should be observed:

1. Amounts owed to creditors other than partners;


2. Amounts owed to partners other than for capital and profits (i.e. partners’ loans to
the partnership);
3. Amounts owed to partners as capital;
4. Amounts owed to partners as profit not currently closed to partners’ capital
accounts.

PROB. 1-70 Suggested answer: (d) Safe payment computations

A lump-sum liquidation is one in which all of the assets are sold in bulk and all of the
creditors’ claims are satisfied before a single liquidating distribution id made to the
partners. Because assets are sold in bulk, there is a tendency to realize greater losses
than if the assets were sold over a period of time. Therefore, payments to partners
should be distributed safely.
PROB. 1-71 Suggested answer: (b) Balances of partners’ capital accounts

The four basic steps to partnership liquidation are:

1. Net income or loss up to the date of liquidation should be allocated to the partners’
capital accounts based on their P&L ratio.
2. The gain or loss realized from the sale of noncash assets should be allocated to
the partners’ capital accounts based on their P&L ratio.
3. Creditors’ claims, including liquidation expenses or anticipated future claims should
be paid.
4. Remaining cash is distributed to partners in accordance with the balance in their
capital accounts, and not the P&L ratio.

PROB. 1-72 Suggested answer: (c)

The provisions that call for the contribution of personal assets to a liquidating
partnership illustrate the characteristics of unlimited liability. However, such personal
liability depends on the legal doctrine of marshaling of assets. This doctrine, which is
applied when the partnership and/or one or more of the partners are insolvent, states
that:

a. Partnership assets are first available for the payment of partnership debts. Any
excess assets are available for payment of the individual partner’s debts, but only to
the extent of the partner’s interest in the capital of the partnership.

b. Personal assets of a partner are applied against personal debts, ranked in the order
of priority as follows: 1.) Amounts owed to personal creditors; 2.) Amounts owed to
partnership creditors; and 3.) Amounts owed to partners by way of contribution.

PROB. 1-73 Suggested answer: (d) 27,500 0 52,000

Cohen Butler Davis


Total
Capital balances 60,000 (10,000) 33,000
83,000
Notes payable to Davis 32,000
32,000
Actual liquidation exps. (7,000) (4,200) (2,800)
(14,000)
Loss on realization
(248,000-218,000) (15,000) (9,000) (6,000)
(30,000)
Addt’l contribution - B 8,500 8,500
Balances 38,000 (14,700) 56,200
79,500
Absorption - Butler (10,500) 14,700 (4,200) -

Cash payments 27,500 - 52,000


79,500

Liquidation expenses and loss on realization are charged against the partners’ capital
accounts. The liability of partners in a partnership is unlimited, thus, the creditors of the
partnerships may go after the partners to the extent of their personal assets. However,
it should be noted that under the doctrine of marshaling of assets, personal assets
should be applied first to personal obligations and any excess shall be applied to the
unliquidated partnership obligation.

PROB. 1-74 Suggested answer: (a) 0

Axel Barr Cain


Capital balances
Before liquidation P40,000 P180,000 P30,000
Loss on realization 4:3:3
(200,000-300,000) 40,000 30,000 30,000
Capital balances
After liquidation P0 P150,000 P0

Since all assets are distributed at one point in time rather than installments, this
represents simple liquidation. As assets are converted into cash, any differences
between the book values and the amounts realized represent gains or losses to be
divided among partners in the profit and loss ratio. Such gains and losses are carried
to the capital accounts. The capital balances then becomes the basis for settlement.

PROB. 1-75 Suggested answer: (c) 195,000 decrease


Nal Lou Gee
Capital balances
Before liquidation P175,000 P125,000 P175,000
Distribution of loss 3:2:5
(P475,000+25,000) (150,000) (100,000) (250,000)
Balances 25,000 25,000 (75,000)
Absorption of Gee (3:2) (45,000) (30,000) 75,000
Capital balances (P20,000) (P5,000) -

Distribution of loss P150,000


Absorption of Gee 45,000
Decrease in capital of Nal P195,000

Note that upon liquidation, all of the partnership’s assets are sold and sufficient cash is
realized to pay all claims except one for P25,000, therefore, the partnership incurred
loss from realization, which will eventually reduce the capital balances of the partners.
In addition, the deficiency of an insolvent partner is simply eliminated by absorption,
thus increasing the decrease in capital account of Nal to P195,000, as shown above.
Incidentally, since Nal and Lou are solvent, additional cash of P20,000 and P5,000,
respectively will be invested by them.

PROB. 1-76 Suggested answer: (b) 170,000 90,000

Given that noncash assets were sold for an amount equal to its book value, therefore,
no gain or loss was realized from the sale of noncash assets. Thus, the partners will
receive an amount equal to their respective capital balances before liquidation.

PROB. 1-77 Suggested answer: (b) 175,000 125,000

Total capital of the partnership (750,000+500,000) 1,250,000


Add liability (Notes payable) 200,000
Total assets 1,450,000
Less available cash 300,000
Loss on realization 1,150,000

Sammy Michael
Capital balances before liquidation 750,000 500,000
Notes payable to Michael 200,000
Total interest 750,000 700,000
Loss on realization (equally) (1,150,000) 575,000 575,000
Capital balances after cash distribution 175,000 125,000

Loss on realization is the difference between the total assets (equal to the total capital
and liabilities) and the amount realized from its sale.

PROB. 1-78 Suggested answer: (a) 136,000

Smith Jones
Capital balances P195,000 P155,000
Smith, loan (Dr) (20,000)
Total interest 175,000 155,000
Loss on realization (60:40)
(450,000-385,000) (39,000) (26,000)
Cash available for partners P136,000 P129,000

Again, since all assets are distributed at one point in time rather than installments, this
represents simple liquidation or total liquidation. The loan account was presented as
an asset of the partnership; therefore, it is a receivable on the part of the partnership,
and eventually, reduces the capital balance of the partner concerned.

PROB. 1-79 Suggested answer: (b) 20,000

Alex Jay John


Capital balances P95,000 P80,000 P70,000
Loss on realization
(150,000-70,000) 2:1:1 (40,000) (20,000) (20,000)
Possible loss
(265,000-150,000) 2:1:1 (57,500) (28,750) (28,750)
Balances (2,500) 31,250 21,250
Absorption of Alex 1:1 2,500 (1,250) (1,250)
Cash distribution P0 P30,000 P20,000

Again, in installment liquidation, to insure an equitable distribution of cash to the


partners, considerable care is required. Usually, the statement of liquidation is
supported by the Schedule of Safe Payments, wherein each installment of cash is
distributed as if no more cash is forthcoming.

Thus, cash is distributed to a partner only if he has an excess credit balance in his
partnership interest after absorption of his share of the maximum possible loss that may
occur, which consists of the assumed unrealizable value of the remaining noncash
assets, ash withheld for payment of anticipated liquidation expenses and unrecorded
liabilities that may arise, and any additional loss that may also accrue to the partners
when a debit balance in any of the capital accounts results from the allocations of
possible loss.

PROB. 1-80 Suggested answer: (b) 25,000

Munoz Nieva Perez

Capital balances
Before liquidation 312,500 107,500 50,000
Loan balances 80,000 25,000
Total interest 312,500 187,500 75,000
Loss on realization (5:3:2)
(900,000-400,000) (250,000) (150,000) (100,000)
Balances (deficiency) 62,500 37,500 (25,000)

The loan balances were presented in the liability section of the condensed balance
sheet, thus, these should be treated as obligations of the partnership and eventually
increase the interests of the partners. Since Perez is the only solvent partner, while
other partners incur no deficiency, Perez will invest additional cash equal to his own
deficiency.

PROB. 1-81 Suggested answer: (c) 5,600

When the liquidation period extend over a long period of time, calculations of safe
payments to partners may become frequent and bothersome such that it may be
desirable to prepare in advance an installment distribution plan, called Cash Priority
Program. This program, which is an alternative to Schedule of Safe Payments, permits
the partners to determine how cash should be safely distributed when it becomes
available. Using this program, the first priority of payments should be provided to the
partner with the highest loss absorption balance. The amount to be paid could be
determined by multiplying the excess loss absorption balance of partner over another
by his profit and loss percentage. The process should be continually done until the loss
absorption balances of the partners are equal, in which case, distribution of available
cash should be made according to their profit and loss percentage.

Since there are three partners in the form (Goh, Kong, and Wei), in the third priority
program of payments, the amounts to be received by Wei is P5,600 (P28,000 x 20%),
because at this point, their loss absorption balances are equal.

PROB. 1-82 Suggested answer: (c) 49,000 0 57,000

Able Baker Chapman

Capital balances 50,000 (32,000) 70,000


Personal assets applied
to capital deficiency
(80,000-50,000) 30,000
Capital balances applied
to personal liabilities
(72,000-60,000) (12,000)
Balances 50,000 (2,000) 58,000
Absorption of Baker (1,000) 2,000 (1,000)
Balances after applying
the doctrine of
marshaling of assets 49,000 0 57,000

The doctrine of marshaling of assets is applied when the partnership and/or one or
more of the partners are insolvent. It provides that partnership assets are first available
for payment of partnership debts. Any excess is applied to individual partners’ debt to
the extent of his interest. While, personal assets are applied against personal debts,
any excess is applied to partnership creditors, then to partner's debit capital balance.

PROB. 1-83 Suggested answer: (c) 320,000


Personal assets 200,000
Add capital credit balances 120,000
Total amount due to personal creditors 320,000

Note that the capital accounts of Morgan and other partners have credit balances;
therefore, the partnership has no deficient partner. Accordingly, there is no possible
absorption that will reduce the capital of Morgan, thus the personal creditors of Morgan
can collect, not only his personal assets but to the extent of his entire capital balance.

PROB. 1-84

a. Suggested answer: (b) 42,000

Total capital before liquidation (40,000+25,000+5,000) 70,000


Less: Cash left for distribution 28,000
Loss on realization of the noncash assets 42,000

Upon payment to partners in the event of liquidation, the amount of cash available for
distribution is always equal to the total partners’ capital.

b. Suggested answer: (c) 17,800

A M E
Capital balances before liquidation 40,000 25,000
5,000
Loss on realization (3:2:1) (42,000) (21,000) (14,000) (7,000)
Balances 19,000 11,000
(2,000)
Absorption of E (3:2) (1,200) (800)
2,000
Cash payment to A and M 17,800 10,200 -

In the event of liquidation, all liquidation expenses and gains or losses from conversion
of partnership assets must be allocated to the partners before assets actually are
distributed to the individual partners. Failure to consider these factors may result in the
premature distribution of assets to a partner. Furthermore, generally accepted
accounting principles states that partners should contribute assets to the partnership
to the extent of their debit balances. However, if such contribution is not possible
because of special personal or legal considerations, the debit balance will be viewed
as a realization loss and allocated according to the remaining partners’ profit or loss
ratio.

PROB. 1-85 Suggested answer: (c) 0 31,000 49,000

Salve Galo Norma

Capital balances before liquidation 80,000 115,000


105,000
Loss on realization (50,000) (5:3:2) (25,000) (15,000) (10,000)
Balances 55,000 100,000 95,000
Less: Possible loss (170,000) (5:3:2) (85,000) (51,000) (34,000)
Balances (30,000) 49,000 61,000
Absorption of Salve (3:2) 30,000 (18,000) (12,000)
Safe payment to partners - 31,000
49,000

To avoid the problem associated with premature payments, installment payments may
be made to partners only after anticipating all liabilities, possible losses, and liquidation
expenses. The allocation of the assumed loss is allocated among the partners
according to their profit and loss ratio. The allocation of the assume loss could produce
debit balances in partners’ capital accounts, and these balances are treated as being
uncollectible. Therefore, the assumed debt capital balances are allocated to those
partners with credit balances according to their profit and loss ratio.

PROB. 1-86 Suggested answer: (c) 410,00 230,000 0

A B C
Capital balances before liquidation 560,000 320,000 40,000
Loss on realization (280,000) (5:3:2) (140,000) (84,000) (56,000)
Balances 420,000 236,000 (16,000)
Absorption of C (5:3) (10,000) (6,000)
16,000
Payment to A and B 410,000 230,000 -
Again, generally accepted accounting principle states that partners should contribute
assets to the partnership to the extent of their debit balances. However, if such
contribution is not possible because of special personal or legal considerations, the
debit balance will be viewed as a realization loss and allocated according to the
remaining partners’ profit or loss ratio.

PROB. 1-87 Suggested answer: (a) 2, 3, 4, 1

Unlike a dissolution where the partnership continues its business purpose, liquidation
results in the partnership's ending or terminating its business. The process of liquidation
consist the conversion of partnership assets into a distributable form and the
distribution of these assets to creditors and owners. All liquidation expenses and gains
or losses from conversion of partnership assets also must be allocated to the partners
before assets actually are distributed to the individual partners. Failure to consider
these factors may result in the premature or incorrect distribution of assets to a partner.

PROB. 1-88 Suggested answer: (c) 40,000

Almond Barney Colors


Total interest 20,000 50,000
90,000
Divide by P&L 20% 40% 40%
Loss absorption balance 100,000 125,000 225,000
Priority 1 - Colors (100,000)
Balances 100,000 125,000 125,000
Priority 2 - Colors & Barney (25,000) (25,000)
Balances (P&L) 100,000 100,000 100,000

Since the question being asked is “one of the partners will get… before any other
partners get anything”, therefore, it is the partner under priority no. 1 (Colors). Colors
shall receive, under priority no. 1, P40,000 (100,000 x 40%)

PROB. 1-89

a. Suggested answer: (b) P70,000 outside liabilities, P30,000 Able


Able Baker Chapman

Total interest 90,000 30,000


50,000
Divide by P&L (equally) 1/3 1/3 1/3
Loss absorption balance 270,000 90,000 150,000
Priority 1 - Able (120,000)
Balance 150,000 90,000 150,000
Priority 2 - Able & Chapman (60,000) (60,000)
Balance 90,000 90,000 90,000
Priority 3 - P&L

Payments by priority: Able Baker


Chapman
Priority 1 (120,000 x 1/3) 40,000
Priority 2 (60,000 x 1/3) 20,000 20,000

Cash Liability Able


Baker Chapman
Total 100,000
Liability (70,000) 70,000
Balance 30,000
Loan - A (20,000) 20,000
Balance 10,000
Priority 2 (10,000) 10,000
Total 70,000 30,000

The Partnership Law provides, that in the event of partnership liquidation, available
cash shall be distributed in the following order: first: to outside creditors, second: to
inside creditors (partners’ loan), and third: to owners (partners’ capital accounts).

b. Suggested answer: (d) Able: 55,000, Baker: 15,000, Chapman: 5,000

Able Baker Chapman


Total interest (excluding loan) 70,000 30,000
50,000
Divide by P&L (equally) 1/3 1/3 1/3
Loss absorption balance 210,000 90,000 150,000
Priority 1 - Able (60,000)
Balance 150,000 90,000 150,000
Priority 2 - Able & Chapman (60,000) (60,000)
Balance 90,000 90,000 90,000
Priority 3 - P&L

Payments by priority: Able Baker


Chapman
Priority 1 (60,000 x 1/3) 20,000
Priority 2 (60,000 x 1/3) 20,000 20,000

Cash Able Baker


Chapman
MV of assets 195,000
Liabilities (70,000)
Able, loan (20,000)
Balance 105,000
Priority 1 (20,000) 20,000
Balance 85,000
Priority 2 (40,000) 20,000 20,000
Balance 45,000
Priority 3 (45,000) 15,000 15,000 15,000
Total 55,000 15,000 35,000
Less: Asset taken
by Chapman 30,000
Balance 55,000 15,000 5,000

PROB. 1-90

a. Suggested answer: (b) 2,592 1,440

Roy Gil Total


Capital before adjustment P94,800 P214,200 P309,000
Accounts receivable (6,667) (13,333) (20,000)
Inventory (7,333) (14,667) (22,000)
Fixed Assets 2,200 4,400 6,600
Total stock to be issued 83,000 190,600 273,600
Common stock (720 x 10) 7,200 7,200 14,400
Preferred stock P100 par P75,800 P183,400 P259,200

Shares to be issued: Roy Gil Total


Common stock 720 720 1440
Preferred stock 758 1,834 2,592

In order to secure the advantages found in the corporate form of business organization,
the partners may decide to incorporate. Upon incorporation, the corporation will act to
acquire the net assets of the partnership in exchange for its stock. The stock received
by the partnership is distributed to the partners in settlement of their equities. The
corporation therefore takes over the assets and assumes the liabilities of the
partnership; the partnership is dissolved and the partners now become stockholders in
the newly formed corporation. The allocated common shares and preferred shares
were computed by dividing the amount of each stock by its respective par value, as
shown above.

b. Suggested answer: (c) 758 720 1,834 720

See above computations in question “a”.

PROB. 1-90

a. Suggested answer: (c) 215,000

Net assets at fair value (P240,000-20,000) P220,000


Less par value of stock issued (1,000 x 5) 5,000
Additional paid in capital P215,000

The incorporation of a partnership results in the formation of a new accounting (and


legal) entity. This means that the partnership must adjust its records up to the date of
incorporation. Among others, the partnership’s books are adjusted to reflect the fair
market value of the partnership assets and the present value of partnership liabilities.
In addition, when capital stock were issued, capital stock account should be credited at
par or stated value, any excess over par or stated value must be credited to additional
paid in capital.

You might also like